SlideShare a Scribd company logo
1 of 126
Download to read offline
A journey of a thousand miles begins with a single step | Yazid Jibrel
Royal Medical Services OBGYN resident
TWITTER@JIBREL MED STUDENTS AND RESIDENTS ARTICLES
Table of Content
1
Table of Content
Table of Content ...........................................................................................................................................1
When did you know? ....................................................................................................................................3
Do you have any suggestions on how to rank residency programs for the Match? ....................................6
When I apply for residency, whom should I ask to write a letter of recommendation? Are these letters
really important? ..........................................................................................................................................8
Don’t Ask for a Letter of Recommendation… Ask for a Strong Letter of Recommendation!.....................10
How Can I Distinguish Myself on Clinical Rotations?..................................................................................11
I am getting ready to apply for residency. Who should I ask to write my letters of recommendation, and
how should I approach them? ....................................................................................................................13
Get the Residency You Want: Tips and Tools: USMLE Scores, Matching Formulas, and More..................17
How Can I Find Love During Training? ........................................................................................................22
Every time I work with a new resident or attending, it seems like the requirements for my clinical case
presentations change. Do you have any tips to help me predict how I should present? ..........................24
It is hard for me as a young student to interview and examine patients. I feel like I am intruding into their
personal world without having a lot to offer them, given my limited knowledge base. Do you have any
advice? ........................................................................................................................................................26
How Should I Deal With Gunners?..............................................................................................................28
Are there any advantages to working outside medicine before you begin medical training?...................30
Once I start practicing in the "real world," can I still change specialties if I feel that I made a mistake? Or
will I be stuck in that career, no matter what?...........................................................................................31
I am starting my residency soon, and I am worried about being prepared. What resources should I bring
to the hospital? How will I know what to do when I'm alone during night call? .......................................37
Twelve Steps for Choosing a Specialty........................................................................................................40
How Should I Choose a Medical Specialty? ................................................................................................42
I'm really worried that maybe I shouldn't be a doctor. What should I do?................................................44
Loving What You Do....................................................................................................................................46
Top 10 Reasons Why I Love My Job............................................................................................................48
How to be a Good Intern ............................................................................................................................50
Who Should Be a Doctor?...........................................................................................................................51
Developing Empathy in Future Physicians..................................................................................................52
In what order should I schedule my clinical rotations? Does it depend on the specialty I am choosing?.56
I would like to be a research assistant or pursue my own research project. How can I find a suitable
professor to work with, and how should I approach one?.........................................................................58
Table of Content
2
Taking Time Off Between Medical School and Residency..........................................................................60
Resume Design for Success.........................................................................................................................62
I'm thinking about doing an elective away from my school during fourth year. What do I need to do to
arrange that? ..............................................................................................................................................64
Can You Learn How to Break Bad News?....................................................................................................67
Are Board Scores Important When Applying for Residency? .....................................................................70
How Can I Increase My Confidence in Doing Physical Exams?...................................................................72
I didn't enjoy my third year of medical school and now I'm not sure if I want to do a residency anymore.
What should I do?.......................................................................................................................................74
How Can I Be Successful on the Job?..........................................................................................................76
How Do You Handle Pre-examination Stress?............................................................................................78
What If I Match a Residency Program I Don't Want?.................................................................................80
Why Doesn't My Resident Trust Me? .........................................................................................................82
What is more important in practicing medicine, the theoretical knowledge of the science or the practical
intricacies and skills?...................................................................................................................................84
How can I remember and integrate the basic subjects of medical school with real clinical practice? ......86
I just finished medical school and am now an intern. How can I already be expected to teach medical
students?.....................................................................................................................................................88
How Can I Predict My Success in the Residency Match?............................................................................90
How Can I Improve My Surgical Skills? .......................................................................................................92
How Can I Find a Mentor? ..........................................................................................................................94
How Should I Choose a Residency Program?..............................................................................................96
What Is the Best Way to Learn Procedures? Doing Them..........................................................................98
Do I Need To Do Research?.......................................................................................................................102
You will Survive.........................................................................................................................................103
Spicing up medical education ...................................................................................................................107
My First Day as a Doctor...........................................................................................................................112
Writing A Medical Case Report.................................................................................................................117
PDAs and Smartphones: Clinical Tools for Physicians...............................................................................120
When did you know?
3
When did you know?
Joshua Batt, Medical Student, Emergency Medicine, 10:19PM Nov 6, 2010
As I entered the gross anatomy lab within my first week of classes -- white-washed walls,
stainless steel dissection tables, and fresh white lab coats moving about the room -- I knew I was
in medical school. The feeling of making progress hit me when I saw the sea of cleanliness, the
closest thing to a sterile environment and the resting place
of our first patients.
That feeling was not to be the last. The next significant
affirmation was getting "pimped" on my first rotation. I
knew then that I was beginning a career of integrative
thinking seasoned with a little medical Trivial Pursuit.
Although not as positive as the former experience, getting
the question wrong was exactly the introduction I
expected from this caliber of education; no matter, I'm in
medical school, right where I want to be.
Such defining moments may have a make or break effect. Whether rocking a board exam or
losing a patient, colorful experiences paint our character and how we will one day practice
medicine. These seem to be the very same milestones that pave our way to graduation. But the
first is where the marathon begins.
I have often wondered what other students define as landmark experiences of entry into their
profession. Is it the endless reading assignments, the account-emptying tuition costs, suturing a
laceration or overwhelming responsibility of caring for the life of another individual? Do you
remember the feeling or event that made you feel like a soon-to-be-doctor or a physician? I
would like to know... when did you know?
A 12-step Program for Step 1?
4
A 12-step Program for Step 1?
Rosalyn Plotzker, Medical Student, Family Medicine, 04:23PM Jul 16, 2010
Hello from the No Man's land, between the boards and third year. I just got back from
post-USMLE rehab.
While I was studying, I found unlimited advice about Step 1 preparation. Mysterious
experts with personal websites weighed in on everything -- what to do early second
year (buy First AID), the weeks before the test (do practice questions until you go
blind), up until Step 1 Eve when you're packing your lunch and resorting to prayer
("just stay calm"... right).
But where were the experts afterward? How do people recover from the inevitable post traumatic
stress? Should you sprint from the Prometric center to the nearest happy hour? Seclude yourself
in your living room and watch Transformers movies? Get on the first boat to the Bahamas?
You can take the med student out of med school. But how much sunburn and bad acting does it
take to get the school out of the student?
My plan was to flee the country. Then, one month before the test, my vacation plans fell through.
I was left with an empty week, and not enough student loan money to finance a trip to, well,
anywhere. I needed something to distract me, a place totally strange, preferably beautiful, and
cheap.
A month later I was at the Tuscon international airport. I signed up to volunteer in the desert.
Like a lot of students, I'm a sucker for volunteer work. First of all, it's free (or there's a small
donation, way less than a week's vacation). Second, there are no decisions. Why fumble through
a guidebook? Third, other volunteers tend to be good company. No risk of a beach full of whiney
high maintenance tourists.
Besides, I had my personal reasons for volunteering. I wanted to use my body to work, rather
than my mind. I wanted to be with people who did not know me as a med student. Lately there
had been moments when I wondered, "Has medical school taken out my soul and replaced it with
a pathology textbook?" Maybe being with strangers could answer that. And, like most med
students, I wanted the heat to evaporate memories of Goljian Lectures, First AID, Kaplan books,
USMLE World, BRS - I'll stop there.
A friend told me about No More Deaths, which is a non-profit that provides medical aid to
migrants crossing through Sonora, especially people who are severely dehydrated or injured. As
part of the desert team, I hiked through trails, looking for people who needed help. We carried
food, water, and medical supplies in our backpacks.
The boiling point for Step 1 is 108 degrees Fahrenheit. That's about when it evaporates.
My team was wonderful. The landscape was unreal. But, by the third day my body was as
exhausted as my mind. And, rather than hide my career path, I blurted out obscure medical facts
at any opportunity. "No, I haven't seen any scorpions around my tent. Hey, did you know that
scorpion bites cause pancreatitis?" So much for taking the student out of school. I couldn't stop.
Keep reading. The silver linings are much more impressive than this story's clouds.
A 12-step Program for Step 1?
5
Obviously, this group deals with a sensitive political issue, especially given the controversy over
SB1070 right now in Arizona - not to mention the general attitudes towards immigration in the
southwest. My personal feelings are that it's a good idea to help migrants if they need it, because
it's a good thing to help anyone if you can. I'll leave the politics there for now.
I will add that I was originally drawn to medicine because of health care activism work that I did
in college. When I decided to be a physician, I imagined myself working in third world
countries, volunteering for Doctors without Borders, or maybe starting my own clinic. (Who
didn't?)
I hardly had time for any of it in med school. There was barely time to get through the
coursework. I treated multiple choice questions, not patients. No wonder the soulectomy-
textbook-transplant.
But, while I was in Arizona, I remembered the potent excitement of my career: the rush of
treating someone who wouldn’t have been okay otherwise. The thrill of trying to heal someone.
It knocked the Step 1 right out of me.
Someday I will write a personal website on how to recover from the boards. It will be a bright
yellow background, and in bold black letters it will say:
Why did you apply to med school?
Okay. So, now go do that.
Do you have any suggestions on how to rank residency programs for the Match?
6
Do you have any suggestions on how to rank residency programs for the
Match?
Response from Daniel J. Egan, MD
Associate Attending Physician, Department of Emergency Medicine,
St. Luke's-Roosevelt Hospital Center, New York, NY
The other day I was interviewing an applicant for a position in our residency program. She told me, "I
think that I know who I want to rank at the top of my list." (This program, of course!). "It's the middle of my
list that I'm having trouble with. Do you have any suggestions?"
It's that time of year. All of you fourth-year medical students are wrapping up or have completed your
interview journey. You may have traveled all over the country. You certainly spent a small fortune for
airfare and hotels. Hopefully you accumulated some frequent-flier miles in the process. You also likely
have sat through 10-15 sessions describing programs, listened to residency directors try to convince you
that their program is the best, and had one too many "night before" social events with the residents in
each program.
So how do you make your list? Is it clear to you which program should be first? Is it less clear to you, like
my medical student, what to do with the middle of your list?
The most important thing for you to take home from this process is that your rank list should be truly that.
You should rank programs in the order in which you would like to match. Many people believe that there
is a way to outsmart the algorithm. Or maybe one program has given you feedback that makes you think
they really want you. At the end of the day, you are dealing with perhaps the most formidable time in your
development as a physician, and you should go where you want to be. So the list should be your own. It
should not be the order that your dean or faculty advisor says it should be (they have their own interests
in mind as well) but the rank order that would make you most happy.
There are a number of factors to consider as you think about each program. The first and foremost is
something that cannot be determined by a formula or list of pros and cons. The question is, Can you see
yourself as a resident in that program? As you have gone through the process, you likely spent a day
somewhere where you thought there was no way you would fit in. Additionally, there were likely others
that left you almost giddy with the feeling that it would be a perfect fit. I still recall the chairman of the
program where I trained meeting with us on the interview day. He casually spoke about a gut feeling of
whether or not the glove fits. There is something important about your gut. Frankly, it is something that
will get you through many difficult decisions in medicine, and choosing a residency is certainly one of
those.
Certain criteria should always be considered when evaluating a residency program: patient population,
tertiary care exposure, clinic experience for the specialties with outpatient components, resident
independence in regard to decision-making, electives, and where graduates go for fellowships or jobs.
Other criteria that should play a role in your decision-making include:
Do you have any suggestions on how to rank residency programs for the Match?
7
Location. This is not insignificant. There are many glamorous cities in the country, but not every city is
made for every person. Do you know anyone in the city that you are considering? Will you have any kind
of support system outside of the hospital? You will immediately establish lifelong friendships during
residency, and there is something exciting about starting over in a new place. But everyone has a
different personality, and some people find it easier if a network already exists during a very challenging
time.
Housing and salary. Is housing provided? If not, will the program's salary allow you to rent an apartment
and still have money left over for your usual expenses? (See our previous columns on the accrual of
credit card debt and how this should be avoided at all costs!)
Are the residents happy? You will hear rumors along the interview trail. Pay attention. Obviously,
rumors are rumors, but hearing that residents in a particular program are very happy is an important piece
of information.
Do you know any graduates from your medical school who are there? We all have a loyalty to our
school. We want the best for students from our school. Often, these residents will be fantastic resources
as you try to get real, honest information.
What is the academic vigor of the program? If you have a long-term goal related to academics, you
want a program that will help foster that. If faculty members are not doing research and publishing, it may
be more difficult for you to find a mentor. What are the daily or weekly conferences like? What are the
academic requirements of the residents?
Teaching environment. With shorter work hours and increased patient volumes affecting all programs,
many residents feel that the clinical teaching suffers. Ask about bedside teaching. The accrediting agency
mandates a core content of lectures and conferences, but it is really the day-to-day operations that help
you develop your clinical skills. You may best experience this with a second look. Show up on a
noninterview day. See how the residents interact with each other and their attending staff.
Finally, I'll share some advice that I got from my med school dean. He told us to put Post-It notes around
our apartments with each program listed in order. Put them somewhere that you will see every day,
preferably multiple times. Believe it or not, that gut will start talking again. Imagine yourself opening the
envelope with each program named inside, and likely you will have a different response to each one.
Looking at this list and thinking about that feeling may help you tweak the order. It certainly helped me
solidify my final list, as I grew more excited about the prospect of my first choice. And remember, the
formula should work in favor of you, the student. Don't try to play a game. Do not shorten your list
because you think you are guaranteed a spot based on program feedback. Take a phone call or email
from a program with a grain of salt. Do what YOU want to do with your list!
When I apply for residency, whom should I ask to write a letter of recommendation? Are these letters
really important?
8
When I apply for residency, whom should I ask to write a letter of
recommendation? Are these letters really important?
Response from Daniel J. Egan, MD
Associate Attending Physician, Department of Emergency Medicine,
St. Luke's-Roosevelt Hospital Center, New York, NY
As someone who routinely reviews applications to our residency program, I can tell you that letters of
recommendation are very important to a successful match. The initial screening process typically involves
board scores, rotation grades, medical school grades, and Dean's letter summary statement. However,
after an applicant has made that cut, letters of recommendation play an important role in the evaluation
process and truly give insight beyond the standardization in the rest of the application.
Several approaches to letters of recommendation can be taken. The most obvious person to ask for a
letter is a mentor who knows you well. If you have worked with a mentor on a research project or have
formed a relationship outside of a rotation or the classroom, this person should write a letter on your
behalf. Most likely, his or her letter will provide more personal insight about you as an individual and
capture the reader's (i.e., the interviewer's) attention. I often pull interview questions from these letters to
ask about the mentorship or project on which a student worked.
The second most obvious “letter writer” for you should be someone involved in a position of leadership in
the clerkship of your specialty choice. Hopefully you have done well in the clerkship of your future
specialty, and you will need to have someone from the faculty speak on your behalf. If you do not include
a letter from someone in your specialty, it will be a glaring red flag to those of us reading through your
application. In fact, some specialties have requirements about a certain number of letters from staff in
your specialty. Be sure to find this out. Additionally, some specialties (such as emergency medicine)
require a specific form letter; obviously, it is important to make sure you follow these guidelines.
The question becomes, Who should write your letter within a given rotation? A letter from a resident is not
an option. A brand-new attending in his or her first year out of residency may also not be the ideal
candidate. It may be to your advantage to approach someone who is more senior within the department.
Ultimately, the content of the letter is important, but supportive statements from a chair, associate or full
professor, or residency director will be taken seriously. Additionally, many of these faculty are nationally
known, and the reader next to me may know them personally or by name, giving more credence to the
letter.
Faculty members are frequently asked to write letters. People who routinely write letters include the
medical student director for a rotation and members of the residency leadership. Those of us reading a
letter look for certain phrases and comments. If a faculty member is not familiar with letter writing, he or
she may not include these statements and it may be hard for an interview committee to interpret the
letter. A medical student clerkship director will also probably include statements or at least a summary
from multiple faculty members and give a consensus opinion. One individual faculty member can only
When I apply for residency, whom should I ask to write a letter of recommendation? Are these letters
really important?
9
speak for himself or herself. Another tip: if you are unsure of a person's opinion of you, avoid asking him
or her to write a letter. Clearly, you do not want a “bad” or even mediocre letter in your file.
Some tips for deciding whom to ask:
 Speak to upperclassmen who have completed the rotation. Some faculty members in your school
have letter-writing reputations.
 Consider your approach to the faculty member. I have been asked explicitly, “Do you think you
would be able to write me a strong letter of support?” Many faculty will tell you that they are not
comfortable writing a letter even without this specific question, but it is always hard to say "no" to
someone. However, if you deliver this more directed question using the adjective "strong," it gives
someone an out to say "no" if they do not think they can write you an adequate letter.
 Ask the residency director at your school for advice. Tell the residency director whom you are
considering asking to write letters for you. He or she may steer you in another direction if that
faculty member may not be the best option.
 Speak openly with your Dean of Student Affairs. The medical school is acting in your best
interest, and the leadership wants students to be successful and match at competitive programs.
One last piece of advice: Ask for your letter as soon as possible after your clerkship, when you are still
fresh in the mind of the faculty member. Also, remember that every letter you collect does not ultimately
need to be included in your application. Better to have more than you need than not enough.
Don’t Ask for a Letter of Recommendation… Ask for a Strong Letter of Recommendation!
10
Rules of the Road
Don’t Ask for a Letter of Recommendation… Ask for a Strong Letter of
Recommendation!
Joel Schofer, MD, Emergency Medicine, 08:20AM Aug 11, 2010
Students often approach their faculty and teachers and ask, "Can you write me a letter of
recommendation?"
The literal answer to this question is always yes. They "can" write you a letter of
recommendation. Anyone can.
The implied meaning of this question is, "Can you write me a STRONG letter of
recommendation?" No one wants a letter of recommendation that is anything but strong. A
mediocre or poor letter of recommendation can be the kiss of death to any application, but, sadly,
unless you ask about the strength of their letter, a less than stellar letter is what you may wind up
with. And because of the privacy of most letters of recommendation, you may never even find
out about it!
I encourage all students to ask their faculty for exactly what they want, asking, "Can you write
me a strong letter of recommendation?" (or some other similar variant). You may be surprised
by the answer to this question. Some students will be shocked when a supportive faculty
indicates that they could write a letter, but that it wouldn't be all that supportive. It is much
better to know this, however, before a weak letter is written and submitted on your behalf.
In summary, don't ask for a letter, ask for a STRONG letter. If you can't get one, find someone
else to write you the letter.
How Can I Distinguish Myself on Clinical Rotations?
11
How Can I Distinguish Myself on Clinical Rotations?
Response from Graham Walker, MD
Resident, Emergency Medicine, St. Luke's-Roosevelt Hospital Center,
New York, NY
Here are 10 tips to help you earn recognition as an outstanding member of the clinical team:
1. Be enthusiastic. Energy is contagious, and we all need more of it in medicine. Bring it to your
rotations! Even if you're not interested in, say, surgery, look for a particular aspect that you do find
interesting. Help out where you can. Volunteer to do more. It goes a long way.
2. Avoid complaining. As hard as you're working (and I'm sure you are!), your residents are working
even harder (or at least have more responsibilities and are balancing more spinning plates). It's fine to
join in when people are venting about how big your team's census is getting, but you won't be winning
any points by saying how rough your job is. (We all know scut sucks, but if you don't do it, your resident
will be doing it for you.)
3. Anticipate tasks that need to be done. Once you've spent a week or two on a service, you can start
to see some patterns: calling for a nutrition consult, getting social work involved, collecting a patient's list
of home medications, and requesting the old chart. Surprise your resident by thinking ahead, and you'll be
at the head of the pack.
4. Take ownership of your patients. The point of clerkships is to prepare you to be a good clinician and
a good resident. Write a good history and physical ; get your notes ready for rounds; and be the first one
to know about any changes, such as a consultant's recommendations or the medications added while
your team was post-call.
5. Help your intern or resident stay informed. Write good patient notes and provide frequent updates.
Your residents are ultimately responsible, so let them know when laboratory results are back or if the
radiologist just called with CT results, for example.
6. Ask questions. This is huge! It's your responsibility to learn patient management, so if you're not sure
why your team is ordering a test or giving a certain medicine, ask. It will show your interest and allow your
residents to do some teaching (and for most of us, it's enjoyable to pass on knowledge to someone at a
stage of training). Just make sure to raise your questions at the right time: Asking to see the ligament of
Treitz while your resident is trying to get control of a bleeding surgical field will not win you any points.
7. Don't lie. If you don't know the answer to a question, go ahead and say, "I don't know, but I will find
out." This is especially true if you're asked about patient-related information; if you forgot to listen to the
lungs, just admit it. It's much better than saying they were normal and then finding out the patient has
rales and has been fluid overloaded all night.
8. Be nice to nurses and other staff. This should be as obvious as the Golden Rule, but sometimes
students can overlook it. Be the first to introduce yourself, say "please" and "thank you," and try not to
leave a mess. Behind any excellent doctor is a bunch of excellent nurses, no doubt.
How Can I Distinguish Myself on Clinical Rotations?
12
9. Show up early. Or at least be on time. (See "Avoid complaining," above.) Your interns and residents
on the team are getting even less sleep than you, and they are still making it in!
10. Come up with your own patient assessment and treatment plan. There's no better way (besides
maybe #6, above) to show your colleagues that you're thinking and learning. You'll sometimes be wrong,
but you're supposed to be wrong, right? Otherwise there'd be no reason to go through medical school and
residency.
I am getting ready to apply for residency. Who should I ask to write my letters of recommendation, and
how should I approach them?
13
I am getting ready to apply for residency. Who should I ask to write my
letters of recommendation, and how should I approach them?
Response from Geoffrey Talmon,
MD
Surgical Pathology Fellow, Department
of Laboratory Medicine and Pathology,
Mayo Clinic, Rochester, Minnesota
As a medical student applying for residency, I found it very uncomfortable to solicit letters of
recommendation (LORs). After all, I needed letters from professor-level attendings who had the potential
to affect my future. On top of that, I had heard that I was not supposed to review the letters before they
were submitted. Questions percolated through my brain, ratcheting up my stress: Who, and how, should I
ask? Would they say "yes"? Did they know me well enough to recommend me? What do these letters
even say? It was not until I was involved as a resident in recruiting other house officers that I truly
understood how LORs are used by programs and what constitutes a good one.
In several ways, the residency Match process mirrors the job application process in the business world.
LORs can be viewed as professional references. As such, they should discuss attributes that are not
explicitly outlined in other application materials. Qualities, such as work ethic, willingness to learn,
enthusiasm, and teamwork skills, give the audience (ie, interviewers and program directors) insight into
how you would function as a resident in their program.
In keeping with this purpose, the individuals who you ask should have worked with you enough to
comment on these points, hopefully in a positive manner. To carry the most weight, this person should be
a faculty member or practicing physician, not a resident. This can prove to be more difficult in certain
specialties. On an internal medicine service, a medical student may work with the same team for an entire
rotation. In my field of pathology, however, I worked with no fewer than 10 attendings during my 1-month
rotation. I ended up having to ask for recommendations from those staff members who I believed were
the best teachers and who saw glimpses of my enthusiasm.
Asking an attending physician to write an LOR can be a daunting experience, especially if it is a person
with whom you have not worked extensively, as is often the case with a department chair. Keep in mind
that most physicians who regularly deal with medical students have experience in writing LORs and
understand what is required. They often will ask for the perspectives of your supervising residents and
other faculty to garner the required information. In fact, you can explicitly suggest this to the person
writing your letter. In reviewing applications, I have seen writers mention that their letter represents a sort
of "consensus opinion," which usually makes that letter at least as strong as any other.
It is most effective to approach your potential writers during a time when you are not on their service; the
earlier in your application process, the better. I found that scheduling an appointment with the staff
physician's secretary was the most practical approach. Having a dedicated time gives you the opportunity
to discuss your goals and plans while refreshing their memory about your time working with them. At the
very least, you should provide a copy of your curriculum vitae and personal statement.
I am getting ready to apply for residency. Who should I ask to write my letters of recommendation, and
how should I approach them?
14
The best LORs that I have encountered integrate specific observations from the writer with information in
the applicant's personal statement. (For example: "On a daily basis I saw the dedication to patients'
spiritual needs that Ms. Smith mentioned in her personal statement as being an important part of her
life.") The more information that you can provide (conversations, application materials, etc), the easier it
will be for your writer to write a strong LOR. Finally, agree upon a "due date" for the letter, preferably
about a month in the future.
Resist the urge to review an LOR before it is submitted; although it may be difficult to accept that this
portion of your application is outside your control, you should realize that residency programs view this
"blindedness" as a control for bias. If you are overly concerned about what someone might write, you may
want to reconsider asking that person to provide a recommendation.
Another important aspect is ensuring that your letters are submitted on time. The writers likely are
incredibly busy individuals, and they may need to be reminded of the deadline; at the same time, you do
not wish to be a pest. If you see that your LOR has not been submitted by the agreed upon date, send a
polite email asking whether the writer needs more information, and then reiterate the due date. This is
where giving your writer ample time is helpful.
In summary, the LOR is a professional reference that is meant to provide residency programs with
additional information to determine whether you will fit in with their team. Strategies, such as setting up an
appointment to converse with your potential writers, providing them with as much information as possible,
and giving them plenty of lead time, can help them write an effective letter that ultimately benefits you the
most.
Quoted from Dr. Louay (Sam’s Friend)
I think this is an important question because in this day and age the LOR are really what make or break
you. Since USMLE scores are no longer reliable to differentiate people (every one scores high), US-written
letters are what will carry you. I will try to break my answer into 3 categories to cover everything you
asked.
Content:
1. LOR has to be “excellent” otherwise it is damaging. After all, you chose which ones to send so
everyone expects them to be at least very good. A half hearted letter is disastrous.
2. The length of the letter is less important than its content. A “great” letter of 1 paragraph is better than
a “good” letter that is a 2-pager. Of course a long great letter is best.
3. The best letter for an FMG is the one that reports outstanding knowledge (better than our
residents/students), being able to fit in the US system and carry patient load, excellent communication
skills (includes language), and excellent rapport with the staff. If there is some “cool” personal info then
it is even better.
I am getting ready to apply for residency. Who should I ask to write my letters of recommendation, and
how should I approach them?
15
4. It is near impossible to discover that a US letter is pre-set (cut and paste) unless the writer made
mistakes. Sometimes you find in the middle of the letter a mention of a name other than the applicant’s.
Or sometimes you find “he” instead of “she”!
5. The shorter the time of contact the less valuable the letter.
6. “Although” can be good or bad base on the context: -“Although he has an accent, his communication
skills r great” is 50-50
- “In spite of different culture and language, he was able to deal with pt well.” is also 50-50.
- “Although the medical system is different the one in Syria, he was able to deal with it like our students
did” is excellent!
-“Although he had some difficulties with English at the beginning, he did very well and improved
throughout the month” is very bad!
Source:
1. The most important factor of the letter is the writer. After all you are taking someone’s word, so
he/she better be credible. Since most PDs will not know the writer, they look at the academic titles and
leadership positions. Someone who is a professor is more credible than an assistant professor (older and
wiser). Someone who is a department chief or a chairman of medicine is highly credible. Someone who
deals with residents and students frequently (residency PD or a clerkship director) has credibility.
2. MD or DO does not make a difference. What makes a difference is academic rank/position.
3. American names are more credible than foreign names in general. A PD may wonder whether the
“foreign” name is a relative or a family friend otherwise. This will be less of a factor if the rest of your
“American” letters are consistent.
4. The institution is very important, the stronger the better. Excellent letters from the Mayo clinic or Duke
are 10 times more important than excellent letters for UTMB (if you are better than their students then it
is good to know what students are they comparing you to?).
5. LOR from outside the US are almost useless unless the program is NOT used to FMGs. Programs who
see FMGs regularly know that these are usually generic letters that have no real meaning. Programs that
seldom see a FMG may take them seriously for lack of experience with them.
6. Programs may call your letter writers if they are having a tough time making a decision how high to
rank you. If they take the trouble of doing so, at least it means they are interested.
General Strategy:
1. The more US letters the better (as long as they are very good). The Dean’s letter is enough from your
school.
I am getting ready to apply for residency. Who should I ask to write my letters of recommendation, and
how should I approach them?
16
2. Inpatient medicine rotations are stronger than elective consultation. Observer as the least significant.
There is no difference in regards to type of elective (except if you have an elective where you mostly
watch, such as cath lab, then it is no good). Inpatient is generally better than outpatient because it is
more likely to be rigorous.
3. In general, send the strongest letters (10/10 or 9/10) better than 7/10 no matter what they cover, as
long as the writers are of the same caliber. A 8/10 from a famous tough guy is better than 10/10 from a
young assistant professor.
I hope this covers the topic adequately. Feel free to post follow up questions if you have any.
Get the Residency You Want: Tips and Tools: USMLE Scores, Matching Formulas, and More
17
Get the Residency You Want: Tips and Tools: USMLE Scores,
Matching Formulas, and More
Introduction
Medical students constantly hear that the fourth year is the best year of medical school. That may be true,
but for most students, fourth year also means figuring out how to navigate the residency match process,
which can seem chaotic and mysterious, full of myths and speculation.
Students basically want answers to some simple questions: What is a particular program like? How
competitive am I for a given program? To how many programs should I apply? What are residency
directors really looking for? And how can I really shine in an interview?
Although there is no one answer to any of these questions, there are experts who can provide some
guidance. At the recent annual meeting of the Association of American Medical Colleges (AAMC), leaders
from 2 schools offered their ideas about how students can be successful residency applicants. The
following tips represent a summary of the insights shared by Paul Jones, MD, Associate Provost of
Student Affairs and Director of Pediatric Otolaryngology and Resident Education at Rush Medical
College, Chicago, Illinois, and Angela Nuzzarello, MD, MHPE, Associate Dean for Student Programs and
Professional Development and Assistant Professor in the Departments of Psychiatry and Medical
Education at the Feinberg School of Medicine, Northwestern University, Chicago, Illinois.
USMLE Scores, Matching Formulas, and More
Characteristics of applicants in the 2005 residency match are outlined in "Charting Outcomes in the
Match,"[1]
a publication of the AAMC and the National Resident Matching Program (NRMP). The report
includes average scores on the US Medical Licensing Examination (USMLE) for applicants in the 2005
match. For some specialties, median USMLE scores and the ranges of scores were quite broad, so
students should use these data only as a gauge of their competitiveness, not as a reason to choose a
particular specialty, Dr. Jones said.
Although some residency programs do not look at applicants' scores on the Step 2 Clinical Knowledge
exam (known as the Step 2 CK), a growing number of programs do want to review these scores and are
putting greater emphasis on students' performance in this area, Dr. Jones said. Students should check
with individual programs to see whether they require Step 2 CK scores before they interview with the
program or complete their rank lists, he advised.
Dr. Nuzzarello recommended that students find a research project in the specialty area that they want to
pursue. Rather than just listing publications or job experience on a résumé, students should present
research projects by explaining their particular roles in the project, as well as describing the skills they
acquired. Thus, even without a long list of publications, students can convey their level of understanding
and proficiency in laboratory skills.
Each residency program has its own "formula" for selecting applicants, and program directors may or may
not share this formula with students. However, students should focus on trying to be themselves, Dr.
Jones and Dr. Nuzzarello agreed, so that both the interviewer and the applicant can see what the other is
really like. Despite what many students believe, residency program leaders can be just as surprised on
Match Day as the applicants; they really do not know who will be filling their residency slots, and they are
Get the Residency You Want: Tips and Tools: USMLE Scores, Matching Formulas, and More
18
eager to impress applicants just as students hope to impress them. In the end, the match process seems
to work, the speakers said, so students should trust the process.
Dr. Jones reviewed studies that looked at how some program directors use data to rank residency
candidates. One survey of program directors in orthopaedics found that the top 10 things they valued
were (in diminishing order):
 the student's completion of a rotation at the program director's institution;
 performance on USMLE Part 1;
 rank in medical school;
 formality and politeness at the interview;
 personal appearance;
 performance on questions of ethics at the interview;
 letter of recommendation from an orthopaedic surgeon;
 membership in the American Orthopaedic Association;
 medical school reputation; and
 the dean's letter.[2]
Dr. Nuzzarello identified red flags that dissuade residency directors:
 Failure of USMLE Step 1
 Failure of a core clerkship
 Unexplained leave of absence
 Formal disciplinary action
 "Extra" requirements or "extra" time
 Obtuse personal statements
 Negative letters of recommendation
 Falsified or exaggerated entries on the résumé
If, for whatever reason, a student took time off during medical school (even for a research rotation), he or
she should explain this fully, because some faculty will only notice that the student did not graduate in 4
years, Dr. Jones said. Any student with one of the above "red flags" in their application should be
prepared to address it during an interview. Dr. Jones added that, in his experience, students who are
unsuccessful in the residency match are unaware of how they measure up against other applicants.
Get the Residency You Want: Tips and Tools: USMLE Scores, Matching Formulas, and More
19
Students who have a red flag, but who also realize it and address it in their interviews, seem to do better,
he said.
Interview Tips and Tools
Dr. Nuzzarello provided an overview of what students need to know and do before they begin the
residency interview process. The following lists summarize her recommendations.
Interview tips:
1. Know what is unique about the program
2. Be aware of any recent changes in the program or department
3. Learn about the department's faculty and ongoing research projects
4. Use the Internet to learn about the program and hospital
5. Speak with alumni
6. Study the geography of the program, ie, remote clinical sites
7. Research the community
What to bring to the interview:
1. Directions and contact phone numbers
2. Any information you have on the program
3. Multiple copies of your CV
4. Copies of your papers and published articles
5. Portfolio, pen, pencil
6. Small suitcase, if traveling
7. Any updates to the Medical Student Performance Evaluation (MSPE) or transcript
8. An interview checklist
Tips for the interview:
1. Start with a handshake and make good eye contact.
2. Don't be afraid to smile.
3. Be aware of your body language (eye contact, position in chair).
4. Show enthusiasm for the program, no matter how interested you are.
Get the Residency You Want: Tips and Tools: USMLE Scores, Matching Formulas, and More
20
5. Answer each question and then pause; know when to stop.
6. Pay attention to interviewers' cues.
7. Be wary of programs that do not allow you to interact with residents.
Questions and topics to anticipate in the interview:
1. Tell me about yourself.
2. What would you do if... (ethical dilemmas).
3. What attracts you to this specialty?
4. Describe the most interesting case you've seen.
5. Why should we choose you over other candidates?
6. Where do you see yourself in 10 years?
7. What are the biggest issues facing physicians today?
8. What is your greatest weakness?
Questions to ask the faculty (interesting questions will make you stand out, as faculty often hear
the same questions all day):
1. Are any program changes anticipated?
2. What percentage of residency graduates have passed the specialty board exams on the first try?
3. Do you assist graduates in finding a job?
4. What do you feel are the strengths of this program?
5. What do your graduates do after residency? (what types of jobs, where they are located)
6. What is the balance of didactics and clinical work?
7. How much contact do the residents have with faculty?
8. Are there opportunities for research? (only if you are interested)
9. Are there nonclinical requirements (research, writing, administrative tasks)?
10. Is there elective time?
Questions to ask residents:
1. What aspects of the program need improvement?
2. What is one thing that you would change about this program?
Get the Residency You Want: Tips and Tools: USMLE Scores, Matching Formulas, and More
21
3. How much contact do the residents have with faculty?
4. What do residents generally do with elective time?
5. How are residents evaluated?
6. Are you receiving adequate feedback?
7. Are there international opportunities?
Questions not to ask:
1. What is the salary?
2. What is the benefit package? (This should be in the information packet.)
3. How much vacation do I get?
4. When can I start moonlighting?
Finally, always treat the residency coordinator well. Your interview starts with the first contact you have
with the program, and this is often with the coordinator. Many program directors ask the coordinators
what they thought about applicants, so it pays to be nice to them.
Also, students should follow up interviews with thank-you notes. For interviews with multiple faculty
members, a single thank-you note to the program director will suffice. In fact, this is an opportunity to
remind the interviewer about something interesting about yourself.
In summary, the presenters emphasized that this is a match process where both sides are trying to get to
know each other. Students cannot predict what program directors are looking for, so they should follow
the above recommendations but they should also try to be themselves. This way, programs can
determine which candidates are the best fit for them.
How Can I Find Love During Training?
22
How Can I Find Love During Training?
Question
Developing a romantic relationship seems very difficult during medical training. Is there anything I can do
to improve my prospects?
Megan Fix, MD
Attending Physician, Maine Medical
Center, Portland, Maine
Although I wish it were not so, there is no "love potion number 9" to magically establish a relationship.
Maintaining strong relationships with others, however, is vital to your well-being during training. Can you
find someone during this time? Yes. How do you find someone? My advice is to start by looking inward
and be honest and true to yourself. Once you build a good relationship with yourself, do the same with
your friends and family, and when those are strong and healthy, you will best be able to find and hold
onto a significant other. There are many books and articles that focus on the "medical marriage" and on
nurturing an established relationship, but in the interest of space, this article focuses on finding love
during medical training.
First, it is so important to love yourself. It seems simple, but with the long hours, negativity, and grueling
demands of training, it is hard to remember. Some of us look outside ourselves to fulfill what we need
inside. We are smart, accomplished physicians but we still feel small. We look for good grades and good
comments from superiors to make us feel better inside. This need for approval can spill over into our
relationships. The fact is that you are good enough just being you. If you really believe that, then others
will, too. If they do not agree, or try to put you down, then they are not worth your time. If you are happy
with yourself first, you will have so much more to give and to gain when you are in a relationship with
another.
Once you learn to truly love yourself, the next step is to be honest about what you want and need. This is
important in residency not only because you have limited amounts of free time, but also because it is the
time when most of us are emotionally maturing. Is it important to share religious beliefs with your partner?
Is it important to share outdoor interests with them? Is it important to share interests in politics? Whatever
it is, give yourself the liberty to trust what you need. When you think of a golden weekend, what is it that
you want to do? Run, write, paint, or sit in a cafe? Whatever it is, make sure that you nurture your core
values.
Next, share these core values with others. Start with friendships. If you value cycling, for instance, find
others who do as well. Join a club. Your happiness in doing things that you love will show, and it will spill
out to those around you. If you love music, find a live music spot and bring along a medical colleague. Not
only will you strengthen your friendship, but perhaps you will find a potential significant other. Avoid going
out just to "find someone," because it often is superficial. Spend time being you and doing the things that
you enjoy, and kindred spirits will be nearby. In his handbook on residency, Dr. Lyle Victor writes, "a
How Can I Find Love During Training?
23
happy marriage involves the definition of mutually agreed upon lifestyles and goals.[1]
" Establish goals for
yourself; share them with someone else; and they will be much easier to embrace in a long-term
relationship.
People often debate dating someone who is "in medicine" or "not in medicine." There are pros and cons
to each, but no right answer. Some will say, "I only meet people at the hospital" in defense of the former.
Others will say, "We always talk about medicine; I need someone who lives in the real world" to support
the latter. My advice is, again, be yourself and find people who are in line with what you like -- no matter
where they work.
What about Internet dating? I am a big advocate of this for medical residents. Three of my fellow
residents have found their partners online and are amazingly happy. Internet dating can reduce the time
you have to spend searching for someone, and it can help you find like-minded people in your area. The
only pitfall to avoid is getting too overwhelmed. Remember that it is okay to be selective with your time.
Your priority should be first to yourself and your patients, then to your dating. Your time is precious, and if
you are not connecting with someone, politely say "no, thank you."
Once you do find someone, however, whether they are in medicine or not, it is important to communicate.
Focus on the positive aspects of being a physician. Share your passions and remember why you went
into medicine in the first place. There are many negative aspects of training, but try not to dwell on those
when you are meeting someone. For example, if you had a difficult patient encounter before a date, feel
free to share what bothered you but don't dwell on the medicine aspects. Emotions are common to all of
us. Share your passions and joys.
Finally, be honest about your schedule. It is hard for anyone to understand the hours we put in during
medical training. Do not try to meet someone at 6:00 pm when you know that you may not be out until
6:45. Just as you nurtured your relationship with yourself, you must nurture your relationship with another
person. Even when you come home and are tired, remember that a healthy relationship is a 2-way street:
You give, and they give. Feel good about giving. In his article on the well-being of physician relationships,
Dr. Michael Myers states that "all relationships require care, patience, and nurturing.[2]
" Start by nurturing
yourself, and that foundation will serve as fertile ground for nurturing your relationship with your partner.
Medical training is a defined period of time: for most of us, somewhere between 7 and 11 years. That is a
long time to be down on yourself for not having a relationship, or for being in the wrong one. So, do what
you can; enjoy who you are; share with those around you; and be excited about the potential of true
happiness with someone who shares the good parts of you.
Every time I work with a new resident or attending, it seems like the requirements for my clinical case
presentations change. Do you have any tips to help me predict how I should present?
24
Every time I work with a new resident or attending, it seems like the
requirements for my clinical case presentations change. Do you have any
tips to help me predict how I should present?
Response from Thomas E. Robey, MD, PhD
Resident, Emergency Medicine, Yale-New Haven Hospital, New
Haven, Connecticut
I've been mulling over my response to this great question for several weeks, only because there are so
many potential answers but also a number of pitfalls. So when my new senior resident on an off-service
floor month gathered our team before rounds this morning, the answer jumped out at me: Just ask! In
about 5 minutes, I knew how to structure my presentations for the rest of the month.
It's natural to be apprehensive about rounds, and the guessing game about oral presentations doesn't
end when you graduate from medical school. As long as you are junior to another doctor (in effect for
your foreseeable future), you will need to adjust your presentation to accommodate your supervisor. And
isn't that true when you talk to anyone? You'd use different language, mannerisms, and sentence
structures for outreach at a grade school vs when you defend your PhD dissertation, right? In the same
way, you can expect to use different styles on surgery or medicine clerkships.
That said, I had a tough time with rounds in my third year of medical school. This was usually to my
detriment, as evaluation comments would come back to the tune of, "Thomas understands the clinical
information but struggles to present it in a concise manner on rounds." Of course, one attending
complimented me on my monologues as she chastised a senior resident trying to interrupt me. She said,
"I find Thomas' presentations refreshing. They remind me of reading Faulkner." I'm not sure whether that
was actually a compliment.
But I digress. The only contact you may have with your attending is on rounds, so it's important to make
sure you're giving the presentation that he or she wants to hear. When you start a clerkship, ask your
resident what he or she expects and what the attending prefers. Be specific! Here are some concrete
questions to ask your senior resident:
 What do you expect from my clinical presentation?
 Do I present my plan by problem or by system?
 Am I shooting for 10 minutes, 3 minutes, or 3 sentences?
 Should I expect interruptions?
 What does Dr. Attending like and hate in a presentation?
 Does the team generally read from notes?
 How are my presentations going?
 Could you meet me 15 minutes before rounds to go over my presentation?
I found that last one the most useful. Usually interns are too busy to help out before rounds, and the
senior resident is better situated to offer feedback. Are you still unsure of the general skeleton of your
Every time I work with a new resident or attending, it seems like the requirements for my clinical case
presentations change. Do you have any tips to help me predict how I should present?
25
presentation? At some point in medical school, you should have received coaching for this. If you need
more, the UCSD School of Medicine has an excellent online guide for comprehensive oral case
presentations. As a medical student, you are allowed to err on the side of too much information, and as
your experience grows, your presentation will shrink. Soon enough, you'll be an expert yourself!
It is hard for me as a young student to interview and examine patients. I feel like I am intruding into their
personal world without having a lot to offer them, given my limited knowledge base. Do you have any
advice?
26
Question
It is hard for me as a young student to interview and examine patients. I
feel like I am intruding into their personal world without having a lot to
offer them, given my limited knowledge base. Do you have any advice?
Response from Daniel J. Egan, MD
Associate Attending Physician, Department of Emergency Medicine,
St. Luke's-Roosevelt Hospital Center, New York, NY
First of all, I commend you on recognizing a level of discomfort with what you are doing. In medicine, we
tend to normalize things that would not be considered normal anywhere else. Early on in your training,
you are more likely to recognize and be uncomfortable with what the rest of the world may consider
"abnormal."
There is a unique and relatively rapid transition that occurs as you become a medical student. Most
schools have now introduced a clinical exposure to some degree early in training. In the setting where I
work, first-year students shadow me and begin interviewing patients as part of the Fundamentals of
Clinical Medicine course. Just a short time ago, these students were regular college students; suddenly,
after putting on the white coat, they have taken on a very different role.
Your statements raise several issues. The first is the basic difficulty in examining and interviewing
patients. You are suddenly prying into the history (both previous and present) of these individuals. We
frequently label patients by their diagnoses (eg, the "chest pain" in room 2; "stroke" in room 7). However,
behind each diagnosis is a person with a medical history, a social history, and family members who care
about them. You are expected to ask questions about all of those things, and in return, you have an
expectation that the patient will share his or her answers with you. But what if the patient has a question
for you about his or her illnesses or history? You should understand that it is okay to tell the patient that
you do not know. A good rule of thumb for your entire career in medicine is that making something up is
never the right thing to do. Explain that you are still in the process of learning, and you will find out and
get back to your patient.
You use an interesting word when you say you are intruding. In some ways, your interview is an intrusion
into intimate and private details of a person's life, but certainly your physical examination can be intrusive.
Reading this made me immediately remind myself that this is truly what the privilege of being a physician
(or physician in training) is all about. We put on a white coat, or merely introduce ourselves as doctors or
student doctors, and patients share very private details with us. Additionally, they allow us to examine
them to try and determine the etiology of their symptoms. It truly is an honor and a privilege, and
sometimes it takes the novice to remind us of that. You are correct. It is an intrusion. However, the bigger
picture is that the intrusion is necessary to get at the greater good, which is the successful diagnosis and
treatment of your patients.
It is hard for me as a young student to interview and examine patients. I feel like I am intruding into their
personal world without having a lot to offer them, given my limited knowledge base. Do you have any
advice?
27
So, let me congratulate you. You are truly a grounded person who has not yet forgotten the humanity in
medicine. Please try to remember this feeling from time to time as you move forward in your career.
Although we play a certain role as physician, our patients are people as are we. The physician who does
not forget his or her roots, recognizes each patient as a unique individual, and remembers that this is an
honor will serve his or her patients better.
With time, your feelings will change. Some things will always seem a little strange but likely more
"normal." This is neither good nor bad, just part of the process. I hope you can remember asking this
question in the years to come.
How Should I Deal With Gunners?
28
How Should I Deal With Gunners?
How do you deal with a classmate who is a gunner? Is it something that I even need to worry about?
Response from Sara Cohen, MD
Fellow, Department of Physical Medicine and Rehabilitation, Harvard
University, Boston, Massachusetts; Fellow, Department of Physical
Medicine and Rehabilitation, VA Boston Healthcare System, Boston,
Massachusetts
During one of the first anatomy labs in medical school, I was approached by the professor. After he
examined our dissection of the abdominal cavity, he reached his gloved hand deep into the cadaver and
spoke to me for the very first time as he pointed at a blood vessel. "Sara," he said, "can you tell me the
name of this artery?"
I was pleased because I knew this one. I opened my mouth to answer the question, but before I could get
a word out, my lab partner piped up, "The gastroepiploic artery."
I was shocked. The professor had clearly addressed the question to me. He used my name and he was
looking right at me. There was no way that could have been misunderstood. Why would my lab partner
yell out the answer to a question that was obviously not directed at her before I even had a chance to
speak up?
It turned out that this was my first experience with a "gunner."
The definition of "gunner" varies depending on who you talk to, but it generally refers to an especially
competitive medical student. Every medical school class has at least 1 gunner, and usually many more
than that. Some people use the term to refer to students who study much more than average and are
especially concerned with grades. However, the term may also be used to refer to medical students who
exhibit behavior that is either borderline unethical or even blatant cheating.
Gunner behavior in the preclinical years that is borderline unethical includes (to name a few) hoarding
study materials, making comments in front of professors that are meant to make the gunner look smarter
and other classmates look unprepared, and dominating small group discussions. More malignant gunner
behavior includes ripping pages out of textbooks in the library, sending out erroneous study materials to
the class, or even cheating on examinations.
As a medical student, you will almost certainly encounter a gunner classmate at some point. (If you don't,
the gunner might be you!) Many specialties have become very competitive, which puts pressure on
students who want to match in these specialties to be at the top of the class. If a gunner is making you
feel unprepared or making it difficult for you to learn, the easiest approach is to avoid him or her in an
academic setting. If you have a study group, make sure it only includes classmates who are conducive to
your learning, rather than people who are trying to top you or make you feel insecure. Just because a
classmate is your friend, that doesn't obligate you to study with that person as well.
How Should I Deal With Gunners?
29
Another strategy is to set a good example. If you share your study materials with the class, it will
encourage others to do the same. This is a subtle way to let gunners know that information should not be
hoarded, and it is best for everyone to learn the material.
If a gunner is making class very unpleasant by dominating the lecturers' time, and the subtle hints aren't
working, the only option might be to approach him or her and explain your concerns. Most students don't
want to be perceived as gunners, so pointing this out to the student might make him or her change. When
you speak to your classmate, be nice and respectful when you address this issue -- after all, this is
someone you're going to be working with for the next several years and you don't want to make an
enemy.
Finally, if the gunner is outright cheating, it is your responsibility to report this to the appropriate
authorities.
Unfortunately, during the clinical years, gunner behavior often escalates, because grades are largely
based on evaluations from the attending physicians who are observing you. A gunner may try to be the
first person at work every day and the last to leave. He or she may try to leap in and do every available
procedure or surgery, even on a patient who belongs to another student. I've heard of students who
looked up the laboratory results on their co-student's patients to be ready if the other student dropped the
ball.
The best thing you can do in that situation is try to ignore the gunner's behavior and do the best job you
can on your own patients. If you are very diligent and knowledgeable, that will be apparent to your
attendings and residents. However, if the gunner's behavior is very disruptive, the best recourse may be
to confront him or her.
Keep in mind that all attendings and residents were once medical students, and they're often able to
recognize gunner behavior. Although the gunner may impress some attendings, others will be turned off
by attempts at showing off. You may be gratified to discover that your attending dislikes your gunner co-
student as much as you do.
Although gunners are a common occurrence in medical school, there's no reason for them to make your
life miserable unless you let them. Work hard and learn as much as you can and take gunners for what
they are: harmless distractions.
Are there any advantages to working outside medicine before you begin medical training?
30
Question
Are there any advantages to working outside medicine before you begin
medical training?
Response from Sarah Bernstein, MD
Resident, Department of Obstetrics and Gynecology, St. Luke's-
Roosevelt Hospital,
New York, NY
Before I started medical school, I worked as a waitress for a year. It was mainly a way to make ends meet
while I finished my premed requirements, but it turned out to be a very beneficial experience and one that
has definitely affected my career as a physician. Approaching a table of customers is very similar to
approaching patients. You have to read your customers; be approachable, friendly, and professional; and
then do your best to serve them well. You need to work well in adverse circumstances: If the chef forgets
their meal, a free drink is always helpful! You also need to make everything accessible and
understandable to the everyday person.
Back in those days, I often found myself describing the composition of a beurre blanc sauce; now, it is the
warning signs of preeclampsia and preterm labor that I must explain. The endurance developed by long
nights on my feet carrying trays of food up and down stairs certainly helped as well. Perhaps the most
important lesson that I learned was to always check in on your customers. A customer can be content at
one moment and then ready to throw the spaghetti at you the next, just as a patient can move from stable
to crashing in a matter of seconds.
You may ask whether I'm advocating that all medical students and residents join the food service industry
before becoming physicians. Definitely not! I do think, however, that life experiences, regardless of their
relevance to medicine, are very helpful in residency. Older residents who may have worked or traveled for
a few years generally approach residency with a very different perspective; they understand that it is a
very extreme and temporary situation. Many older residents also enter residency with a spouse, family,
and maybe even a dog. They are challenged with budgeting their time and not neglecting their loved
ones. On the plus side, they are often able to maintain a better life balance with the love and support of
their families.
No matter where you are in life, residency will be some of your most challenging years. Once you begin,
there is very little time for more than just medicine, so any life experience that you bring from the years
before training will be incredibly helpful.
Once I start practicing in the "real world," can I still change specialties if I feel that I made a mistake? Or
will I be stuck in that career, no matter what?
31
Question
Once I start practicing in the "real world," can I still change specialties if
I feel that I made a mistake? Or will I be stuck in that career, no matter
what?
Response from Geoffrey A. Talmon, MD
Assistant Professor, University of Nebraska Medical Center, Omaha,
Nebraska
A friend of mine during residency had switched to pathology after being a rural family physician for many
years. When I was a frustrated intern, I often asked him why he had chosen to go back to residency; his
wife had a successful career, his children were all in elementary school, and he had already "paid his
dues" as a resident. He said that in his previous practice, he was confident in handling common problems
but often referred complex and interesting patients to specialists. Over time, he discovered that making
one difficult diagnosis was more fulfilling than treating 10 patients with hypertension. Now in a pathology
practice, he reiterates that although the move was initially hard for him and his family, he has found
tremendous professional and personal gratification in his work and does not regret the switch.
The perceived dogma is that being a physician represents a state of terminal vocational choice. Although
it is not uncommon for residents to change specialties, many doctors believe that it is "too late" to alter
their career path after entering practice, partly due to their substantial investment in training. Few (if any)
residency programs broach the topic with their trainees. Practicing physicians may be reticent to discuss
a partner's departure from their practice, even if the change had nothing to do with group dynamics. A
lack of information perpetuates the notion that doctors are stuck with their initial career choice.
The reality is that changing one's specialty is not unprecedented. In fact, as more Generation X'ers and
Millennials populate medicine (with the increased tendency to job-shop), switching specialties likely will
become even more common. Because there are few studies on this phenomenon and because mentors
may be difficult to find, you may feel as though you are going where no one has gone before.
For obvious reasons, making the decision to try a new field after being in practice is substantially more
complex than when one was a resident. Re-entering training (with the attendant staff-resident hierarchy)
may be less palatable. Relocation to a teaching hospital in a larger city may be necessary, placing strain
on spouses with careers and families with older children. Further, the switch will engender a substantial
salary decrease for several years, which may require special financial planning or a lifestyle change.
Obviously, the concept requires careful discussion with and assistance from your family.
Despite all of these potentially contentious issues, the ultimate factor to consider is your long-term
happiness. Many other physicians have made these situations work, sometimes through their own
creativity.
Once I start practicing in the "real world," can I still change specialties if I feel that I made a mistake? Or
will I be stuck in that career, no matter what?
32
Which new specialty to enter may or may not be obvious to you. Perhaps you have always had a "second
choice" in the back of your mind since medical school. Regardless, it is important to consider several
issues: What is the primary source of dissatisfaction with your current specialty? Which aspect of
medicine interests you more? For example (as with my friend), you may find that you enjoy diagnostic
challenges more than urgent care or health maintenance. Perhaps emergent care provides greater
stimulation than long-term treatment of chronic illnesses. Do you want to do more procedures than your
current specialty involves? Perhaps your most serious concerns lie with work hours and scheduling.
Whatever the reason, this decision represents a substantial psychosocial investment, and it is imperative
that the new field you enter is professionally and personally satisfying.
Much like the first time you sought a residency, research is important. Spend time talking to multiple
people from different types and sizes of practices about their experience and views. Specifically,
determine the employment outlook and potential practice settings that would be available when you have
completed training. To this end, it is beneficial to get a feel for the future of the specialty: What issues will
you face in practice (economic liability, political pressures, the impact of mid-level practitioners, etc)?
Length of training is also a consideration. Starting a 1- or 2-year fellowship is obviously less complicated
than starting a 6-year surgery residency. Note that residencies may be willing to grant exemptions from a
preliminary or intern year due to your experience, so be sure to inquire.
When applying to training programs, be prepared not only to justify your decision but also to demonstrate
that you have a good comprehension of the specialty and the previously mentioned issues associated
with switching. Most program directors with whom I have spoken see the benefit and value of having an
"uber-experienced" intern on service and on the educational team. Their primary concern is to maximize
the training of the candidate and their contemporaries. Directors wish to ensure that you are making an
informed, contemplated choice and are prepared to accept being a trainee again. Be certain to explicitly
articulate your reasons for changing, and ask interviewers whether they believe that their program will
provide what you are seeking. This reinforces to each person that you understand precisely what you
want.
One benefit of being a "nontraditional resident" is that you may be exempt from the traditional application
process. This allows you to receive offers and accept a position outside of the Match, which affords a
modicum of flexibility, as it is possible to enter residency at a time other than July 1.
Once in training, continue to critically evaluate the new specialty that you have chosen. Taking into
account the special challenges that are part and parcel of being a resident, is this new path giving you
what you expected? Do not hesitate to discuss any perceived issues with your program director as soon
as possible. Their advice is just as applicable to you as to traditional first-year residents.
In summary, the process of changing specialties after residency is more complex than it is during training,
but it likely will become more commonplace in coming years. Research is the most powerful tool to help
you make informed decisions. The psychosocial and financial ramifications are not insurmountable, and
the increased satisfaction may be worth the cost.
Once I start practicing in the "real world," can I still change specialties if I feel that I made a mistake? Or
will I be stuck in that career, no matter what?
33
Question
I will soon be entering my fourth year of medical school, and I just learned that I've been assigned to a
surgical rotation of which the surgeons are known to have very high standards and the failure rate for
students is very high. However, I've decided not to swap rotations. How should I
prepare myself for it?
Response from Ted Melnick, MD
Attending Physician, Department of Emergency Medicine, North
Shore University Hospital, Manhasset, New York
The surgical culture is very demanding and can intimidate the unprepared student. I remember struggling
through my third-year surgical clerkship and being quite apprehensive of my required fourth-year surgical
subinternship. However, I was surprised to discover that my third-year clerkship had prepared me well for
the subinternship.
As a fourth-year student, the skills that you have gained over the previous year can make you a valued
member of the patient care team. The expectations of a final-year medical student are much different
from those of the inexperienced, timid third-year student. You have a different knowledge base; you
hopefully have some procedural skills; and you understand the workings of the hospital. The mere fact
that you are already thinking ahead and have decided to accept the challenge of a more "difficult" rotation
will put you in a better position for success.
The overwhelming amount of work expected of the surgical resident can be lightened by a helpful,
hardworking fourth-year medical student. If you are able to reduce a resident's workload, he or she will
likely think positively of you and will provide positive feedback to the clerkship director.
Here are a few guidelines to follow in order to survive, pass, and maybe even flourish on your fourth-year
surgical rotation.
First, arrive on time. Surgical rounds occur as a team. If you are late, the whole team will be waiting for
you and will be unable to round on time. In fact, if you arrive a few minutes early every day, you will stand
out as someone who understands teamwork and is there to help.
Second, don't draw negative attention to yourself. Especially in the operating room (OR), don't speak
unless spoken to. Although this sounds authoritarian, remember that surgery requires incredible dexterity
and concentration. Asking a question at an inopportune time in the OR may distract the surgeon at a
critical point in the procedure that he or she is performing. This also holds true on rounds. Time is limited
there, so the team is trying to accomplish as much as possible as efficiently as they can. If you draw
attention to yourself on rounds for anything but contributing to getting the job done, you may leave a bad
impression.
Once I start practicing in the "real world," can I still change specialties if I feel that I made a mistake? Or
will I be stuck in that career, no matter what?
34
Third, help with floor work. An overworked surgical intern will sing your praises to his or her seniors if you
help with some of the floor work, allowing him or her to get home a little earlier or to spend more valued
time in the OR. You may want to read and carry The Surgical Intern Pocket Survival Guide.[1]
This small
reference offers all sorts of pearls that will come in handy while taking care of routine floor work and
charting.
Fourth, familiarize yourself with the book, Surgical Recall.[2]
This book is indispensable. It provides just
about any question that a surgeon has ever used to "pimp" a medical student. Find out which procedures
you will be scrubbing for the next day, and read that section beforehand. You should also review anatomy
relevant to the procedure in your first-year anatomy textbook. You can even go the extra mile by
familiarizing yourself with the procedure in a surgical textbook. To the attending and senior residents, you
will appear prepared, interested, and dedicated to the patient.
Fifth, sleep and eat whenever you can. This statement is true throughout your clinical years in medical
school. However, it is even more important on time-intensive rotations, such as surgery. A little nutrition
and rest can make you much happier and more productive.
Once I start practicing in the "real world," can I still change specialties if I feel that I made a mistake? Or
will I be stuck in that career, no matter what?
35
Question
The hospital environment is so drab and depressing. How can I stay
motivated when I have to spend so much time there?
Response from Daniel Egan, MD
Attending Physician, Department of
Emergency Medicine, St. Luke's-
Roosevelt Hospital Center, New York,
NY
Ahh... We have all been there. My guess is that you are in the midst of your clinical rotations and you feel
like all you do is wake up in the morning, go to the hospital, leave, sleep, and start the cycle again the
next morning. Well, my friend, we have all had those feelings and wondered how to keep on going. There
is no magic answer, but there are a couple of key elements to maintaining your sanity throughout medical
school.
The most important thing that I kept telling myself during my clinical rotations was that everyone in the
class ahead of me got through it. Additionally, many of your classmates who have already completed the
rotation that you are on have survived it. To me, that was motivating: They had endured, and so could I. It
is a little reality check as you try to keep your eyes open during hour 24 in your surgical clerkship.
I also kept telling myself throughout my medical school clerkships -- and especially residency -- that I
could do anything for a fixed period of time. In residency, we changed rotations every month, so I would
repeatedly say, "I can do anything for 4 weeks."
But you are correct: The hospital environment can seem drab and can be depressing. Therefore, it is
critical for you to maintain a life outside of the hospital. We all know that it is possible to eat, sleep, and
dream medical school. There is so much to learn and there are not enough hours in the day. However,
you need to remember that you are a person who had a life and interests before you got there.
I have written about this before, but I often remember something that a dean told us students early in our
education: Even if you spend 24 hours a day studying and trying to learn, you will still never know
everything. So make a schedule. Make an agreement with yourself. Fit outside activities into your day
(even if that means just an hour at the gym each evening). Consider it as mandatory as prerounding on
your patients. Consider it essential to your health and well-being.
There is a certain time of year (coincidentally about the same time as when you submitted your question)
that seems to predispose students and also residents to feelings of "drabness" and depression. You wake
up in the morning and it is dark outside. You leave the hospital to go home and it is dark outside. The rest
of the world has enjoyed a full day of sunlight while you roamed the halls of the hospital and only caught a
glimpse of the sun if you went into a patient's room. One of the coping mechanisms for this is to talk about
it. Go out to dinner with your classmates and share your feelings. Of course, try not to talk only about
work. Sometimes a nice, long venting session can do a lot of good; things are put in perspective and you
recognize that you are not alone.
Once I start practicing in the "real world," can I still change specialties if I feel that I made a mistake? Or
will I be stuck in that career, no matter what?
36
This pattern will continue for the next several years as you go through residency, so developing
distractions and coping mechanisms now will only help later. Perhaps the most important step that you
can take is devoting at least part of your day to a nonmedical activity that you enjoy. This can provide the
motivation for getting through the day. For me, that activity was participating in a music group. Although
we didn't rehearse every day, that 1 night a week of doing something that I love helped me to get through
those very long other 6 days.
Keep a novel at your bedside. Spend even just 20 minutes before going to sleep reading something
completely unrelated to a medical journal or textbook. Make a few phone calls to friends who are in other
fields. Read the newspaper. And remind yourself regularly that there is a whole world out there that you
will soon be able to embrace.
I am starting my residency soon, and I am worried about being prepared. What resources should I bring
to the hospital? How will I know what to do when I'm alone during night call?
37
Question
I am starting my residency soon, and I am worried about being prepared.
What resources should I bring to the hospital? How will I know what to
do when I'm alone during night call?
Response
Sohil H. Patel, MD
Sohil H. Patel, MD, Resident, St.
Vincent's Hospital, New York, New York
Beginning your intern year is daunting. I know of few other times when one's level of responsibility
changes as dramatically as when a medical student becomes an intern. In some respects, the transition
requires an understanding that the first time through many new experiences, you may be slow and you
may make mistakes. There are certainly ways to minimize such mistakes and, most importantly, to learn
from them so that they are not repeated.
As a subintern, I lumbered around in my short white coat with pockets brimming full of "survival guides,"
including my personal digital assistant, a pharmacopoeia, my stethoscope, a reflex hammer, various
scraps of paper and notecards with illegible notes and phone numbers, and maybe a flattened granola
bar or two. Early in my subinternship, I was called to see a patient with abdominal pain and lower
gastrointestinal (GI) bleeding.
My thoughts immediately became as scattered as the contents of my white coat. In fact, my first 3
thoughts were: Is this an emergency? Should I be worried about colon cancer or angiodysplasia? What
are those other causes of lower GI bleeding that I memorized for the boards? As I pondered these deep
questions, my resident thankfully arrived and ordered me to check vital signs and do an examination.
Only afterward did I look through the various resources in my white coat and find at least 3 resources with
the same algorithm for dealing with GI bleeds.
So, my first piece of advice is: know what's in your white coat!
If you buy a survival guide (and I recommend the Washington University series), read through it before
you put it in your white coat. Use only 1 survival guide, and become very familiar with it. Likewise, use
either a personal digital assistant or a pharmacopoeia or an online pharmacology Website (if your hospital
has enough computers), and learn how to use whichever resource you choose.
I also found it helpful to carry a few blank note cards. Every day, I jotted down important phone numbers,
or room codes, or simple instructions on how to get things done in the hospital. My notes were a mess, so
each night I would rewrite them neatly on a new note card.
I am starting my residency soon, and I am worried about being prepared. What resources should I bring
to the hospital? How will I know what to do when I'm alone during night call?
38
Other white coat essentials include a stethoscope, a (functioning) pen light, a prescription pad, your sign-
out, and at least 2 black ink pens. For internal medicine, the Mass General Pocket Medicine book is an
excellent resource. If you can fit a snack somewhere in your pockets, that always comes in handy.
Recognize Emergencies
This is probably the most important skill learned during intern year. When a nurse calls you about a
patient, particularly early in your intern year, you should try your best to always see the patient. See for
yourself what types of calls and complaints are emergent and which are non-emergent.
When you see a patient, the best way to determine whether the problem is an emergency is to talk to the
patient and get a set of vital signs. Then, compare these findings with the patient's baseline findings.
These 2 simple steps will provide critical information about a patient's neurological and cardiopulmonary
status.
Certain nursing calls and patient complaints will mandate that you see the patient. Chest pain, shortness
of breath, acute abdominal pain, hypotension, mental status change, new neurological deficit, GI bleed,
and new fever all qualify. Learn which causes and consequences of such complaints are emergent, and
direct your history, physical examination, and management accordingly. Thus, if a patient complains of
chest pain, your first objective is to rule out a myocardial infarction, pulmonary embolism, or aortic
dissection. Only after doing so should you explore whether the chest pain is actually musculoskeletal in
nature.
Finally, especially early on, accept a low threshold for calling your senior resident about all suspected
emergencies.
Arm Your Brain
The more medicine you know, the more you will enjoy and learn from your experiences as an intern.
Unfortunately, the fourth year of medical school is a well-known cause of brain atrophy. Thus, I found it
helpful to brush up before starting intern year. Review the most high-yield, commonly encountered
diseases in your specialty, and know them well (including presentation, diagnosis, and treatment). You
may also benefit from reviewing school textbooks and study aids.
At a minimum, every new intern should know how to read an electrocardiograph, interpret an x-ray, and
understand a blood gas result.
Prepare Yourself Mentally
Anxiety naturally accompanies unfamiliar situations, particularly when your decisions will affect someone
else's health. Avoid adding more stress whenever possible.
A common stressor is getting yelled at by a superior because you made a mistake. It is important to
recognize 2 components to the message.: The first is the yelling, which is mostly a reflection of your
senior's (probably somewhat dysfunctional) personality; the more this is ignored, the better. The other
component is the actual message, which often contains very useful advice to avoid making the same
mistake in the future. This second component deserves your full attention.
Other ways to avoid needless stress are as follows:
I am starting my residency soon, and I am worried about being prepared. What resources should I bring
to the hospital? How will I know what to do when I'm alone during night call?
39
 Try to build a good rapport with the support staff, and never engage in petty arguments with them;
 Know your limits with difficult patients and recruit the help of social workers when needed;
 Build good relationships with other residents;
 Don't be afraid or embarrassed to ask for help from anyone (especially nurses, who are often
your best friend on a busy call night); and
 Do not constantly remind yourself how terrible an intern's life is. If you direct your thoughts to your
patients and the amount of good you are doing for them, the intern year can be an incredibly
rewarding experience.
Twelve Steps for Choosing a Specialty
40
Twelve Steps for Choosing a Specialty
Question
I need to declare a medical specialty before long, but I have so many conflicting feelings and thoughts
about various clinical areas. How can I make the right choice?
Response from Anne Vinsel, MS, MFA
Project Administrator, Graduate Medical Education, University of Utah
Medical Center, Salt Lake City, Utah
It's time for fourth year students to get serious about choosing their specialty area. Some of you are lucky,
and everything lines up: you know which clinical area interests you most, your board scores and
grades/letters are all in the correct range, and you have helpful professors on your side. For you, it's just a
matter of doing the paperwork on time. You can stop reading here.
But I know there are many others of you out there who aren't sure what specialty to choose. Or, you're
torn between 2 or 3 specialties. Or you know what you don't want but aren't sure what you do want. Or
you know what you want, but aren't sure if your qualifications are strong enough. Read on!
If you're stuck, here's a decision tree to follow:
1. Find or make a list of all the specialties available directly after medical school (ie, skip
fellowships).
2. Cross off the ones you definitely don't want. You don't need a string of reasons beyond the fact
that you simply can't see yourself doing it long term.
3. Perform a Google™ search with the phrase "choosing a medical specialty." When I tried it, I got
about 89,800,000 entries. Set a timer for no more than 1 hour and browse through the first
several pages. Take some of the "what specialty are you?" quizzes. If nothing else, they will give
you some ideas and possibly make you think about specialties you haven't explored. You can
safely avoid making an exact ranking of specialties at this point. Just see which specialties you
seem to be most suited to and which you should rule out.
4. Now, list several specialties you can see yourself doing long term, no more than 6.
5. Research those specialties in your institution. Go to the departments and make friends with the
residency program coordinators. If you haven't already done so and haven't rotated in the
program, arrange to shadow a faculty member for a day. Talk with 1 or 2 residents and check out
the pros and cons of the specialty. Finally, ask the program coordinator if your board scores
would be in a competitive range. Most program coordinators won't share their board score cut-off,
but they likely would tell you if your scores are within range.
6. Narrow your list to 2 or 3 specialties. Now, and only now, talk with family and friends. Tell them
you're thinking of these specialties, and get their opinions. Listen hard, and get them to articulate
the basis for their opinions.
Medscape articles
Medscape articles
Medscape articles
Medscape articles
Medscape articles
Medscape articles
Medscape articles
Medscape articles
Medscape articles
Medscape articles
Medscape articles
Medscape articles
Medscape articles
Medscape articles
Medscape articles
Medscape articles
Medscape articles
Medscape articles
Medscape articles
Medscape articles
Medscape articles
Medscape articles
Medscape articles
Medscape articles
Medscape articles
Medscape articles
Medscape articles
Medscape articles
Medscape articles
Medscape articles
Medscape articles
Medscape articles
Medscape articles
Medscape articles
Medscape articles
Medscape articles
Medscape articles
Medscape articles
Medscape articles
Medscape articles
Medscape articles
Medscape articles
Medscape articles
Medscape articles
Medscape articles
Medscape articles
Medscape articles
Medscape articles
Medscape articles
Medscape articles
Medscape articles
Medscape articles
Medscape articles
Medscape articles
Medscape articles
Medscape articles
Medscape articles
Medscape articles
Medscape articles
Medscape articles
Medscape articles
Medscape articles
Medscape articles
Medscape articles
Medscape articles
Medscape articles
Medscape articles
Medscape articles
Medscape articles
Medscape articles
Medscape articles
Medscape articles
Medscape articles
Medscape articles
Medscape articles
Medscape articles
Medscape articles
Medscape articles
Medscape articles
Medscape articles
Medscape articles
Medscape articles
Medscape articles
Medscape articles
Medscape articles

More Related Content

What's hot

Questionnaire for hospitals 1
Questionnaire for hospitals 1Questionnaire for hospitals 1
Questionnaire for hospitals 1shruthipreetham
 
The Mind Teaches the Brain by Caleb Gattegno
The Mind Teaches the Brain by Caleb GattegnoThe Mind Teaches the Brain by Caleb Gattegno
The Mind Teaches the Brain by Caleb GattegnoEducational Solutions
 
Objective structuredclinicalexam
Objective structuredclinicalexamObjective structuredclinicalexam
Objective structuredclinicalexamHellen Kyakuwaire
 
Guìa de epilepsia
Guìa de epilepsiaGuìa de epilepsia
Guìa de epilepsiaJanny Melo
 

What's hot (6)

2016 book management of ingrowing nails
2016 book management of ingrowing nails2016 book management of ingrowing nails
2016 book management of ingrowing nails
 
Questionnaire for hospitals 1
Questionnaire for hospitals 1Questionnaire for hospitals 1
Questionnaire for hospitals 1
 
The Mind Teaches the Brain by Caleb Gattegno
The Mind Teaches the Brain by Caleb GattegnoThe Mind Teaches the Brain by Caleb Gattegno
The Mind Teaches the Brain by Caleb Gattegno
 
Objective structuredclinicalexam
Objective structuredclinicalexamObjective structuredclinicalexam
Objective structuredclinicalexam
 
Guìa de epilepsia
Guìa de epilepsiaGuìa de epilepsia
Guìa de epilepsia
 
edu
eduedu
edu
 

Similar to Medscape articles

Mapedir interviewers reference manual l 19_may2008
Mapedir interviewers reference manual l 19_may2008Mapedir interviewers reference manual l 19_may2008
Mapedir interviewers reference manual l 19_may2008Prabir Chatterjee
 
Rat Lab Manual
Rat Lab ManualRat Lab Manual
Rat Lab Manualk3stone
 
harvard_med_school_-_stress_management Report
harvard_med_school_-_stress_management Reportharvard_med_school_-_stress_management Report
harvard_med_school_-_stress_management ReportCameron Richards
 
Davison, kindra unit 9 assignment
Davison, kindra   unit 9 assignmentDavison, kindra   unit 9 assignment
Davison, kindra unit 9 assignmentK. D.
 
Healing From Cancer...Insulin Potentiation Therapy
Healing From Cancer...Insulin Potentiation TherapyHealing From Cancer...Insulin Potentiation Therapy
Healing From Cancer...Insulin Potentiation TherapyMedicineOfHope
 
life-after-high-school
life-after-high-schoollife-after-high-school
life-after-high-schoolSamantha Salas
 
Total hip-replacement-guide
Total hip-replacement-guideTotal hip-replacement-guide
Total hip-replacement-guideArun Shanbhag
 
HEALTH SECTOR GENDER TRAINING MANUAL.pdf
HEALTH SECTOR GENDER TRAINING MANUAL.pdfHEALTH SECTOR GENDER TRAINING MANUAL.pdf
HEALTH SECTOR GENDER TRAINING MANUAL.pdfGurumurthy B R
 
Arthritis-Medications-A-Reference-Guide-2015
Arthritis-Medications-A-Reference-Guide-2015Arthritis-Medications-A-Reference-Guide-2015
Arthritis-Medications-A-Reference-Guide-2015Ann-Marie Colacino
 
Tolerance Break
Tolerance BreakTolerance Break
Tolerance BreakL P
 
What it means_to_be_an_ohs_professional_final_final_92_pages
What it means_to_be_an_ohs_professional_final_final_92_pagesWhat it means_to_be_an_ohs_professional_final_final_92_pages
What it means_to_be_an_ohs_professional_final_final_92_pagesDave Collins
 

Similar to Medscape articles (20)

Low lit book_fnl_lr
Low lit book_fnl_lrLow lit book_fnl_lr
Low lit book_fnl_lr
 
Phy active
Phy activePhy active
Phy active
 
Mapedir interviewers reference manual l 19_may2008
Mapedir interviewers reference manual l 19_may2008Mapedir interviewers reference manual l 19_may2008
Mapedir interviewers reference manual l 19_may2008
 
Rat Lab Manual
Rat Lab ManualRat Lab Manual
Rat Lab Manual
 
Rat Lab Manual
Rat Lab ManualRat Lab Manual
Rat Lab Manual
 
harvard_med_school_-_stress_management Report
harvard_med_school_-_stress_management Reportharvard_med_school_-_stress_management Report
harvard_med_school_-_stress_management Report
 
Ganotherapy e-book
Ganotherapy e-bookGanotherapy e-book
Ganotherapy e-book
 
Davison, kindra unit 9 assignment
Davison, kindra   unit 9 assignmentDavison, kindra   unit 9 assignment
Davison, kindra unit 9 assignment
 
Healing From Cancer...Insulin Potentiation Therapy
Healing From Cancer...Insulin Potentiation TherapyHealing From Cancer...Insulin Potentiation Therapy
Healing From Cancer...Insulin Potentiation Therapy
 
life-after-high-school
life-after-high-schoollife-after-high-school
life-after-high-school
 
Total hip-replacement-guide
Total hip-replacement-guideTotal hip-replacement-guide
Total hip-replacement-guide
 
Preparing for Hip Replacement
Preparing for Hip ReplacementPreparing for Hip Replacement
Preparing for Hip Replacement
 
Reverse diabetes2012
Reverse diabetes2012Reverse diabetes2012
Reverse diabetes2012
 
Ibrahim Thesis
Ibrahim ThesisIbrahim Thesis
Ibrahim Thesis
 
HEALTH SECTOR GENDER TRAINING MANUAL.pdf
HEALTH SECTOR GENDER TRAINING MANUAL.pdfHEALTH SECTOR GENDER TRAINING MANUAL.pdf
HEALTH SECTOR GENDER TRAINING MANUAL.pdf
 
Arthritis-Medications-A-Reference-Guide-2015
Arthritis-Medications-A-Reference-Guide-2015Arthritis-Medications-A-Reference-Guide-2015
Arthritis-Medications-A-Reference-Guide-2015
 
Tolerance Break
Tolerance BreakTolerance Break
Tolerance Break
 
Paida lajin self healing
Paida lajin self healingPaida lajin self healing
Paida lajin self healing
 
Breast cancer
Breast cancerBreast cancer
Breast cancer
 
What it means_to_be_an_ohs_professional_final_final_92_pages
What it means_to_be_an_ohs_professional_final_final_92_pagesWhat it means_to_be_an_ohs_professional_final_final_92_pages
What it means_to_be_an_ohs_professional_final_final_92_pages
 

More from Yazid Jibrel

Journal club (Grit and truffle) studies
Journal club (Grit and truffle) studiesJournal club (Grit and truffle) studies
Journal club (Grit and truffle) studiesYazid Jibrel
 
Maternal collapse
Maternal collapse Maternal collapse
Maternal collapse Yazid Jibrel
 
Nausea and vomitting in pregnancy
Nausea and vomitting in pregnancyNausea and vomitting in pregnancy
Nausea and vomitting in pregnancyYazid Jibrel
 
how to write an abstract
how to write an abstracthow to write an abstract
how to write an abstractYazid Jibrel
 
Genetics for mrcog part1
Genetics for mrcog part1Genetics for mrcog part1
Genetics for mrcog part1Yazid Jibrel
 

More from Yazid Jibrel (9)

Journal club (Grit and truffle) studies
Journal club (Grit and truffle) studiesJournal club (Grit and truffle) studies
Journal club (Grit and truffle) studies
 
Maternal collapse
Maternal collapse Maternal collapse
Maternal collapse
 
Laparoscopy
LaparoscopyLaparoscopy
Laparoscopy
 
Nausea and vomitting in pregnancy
Nausea and vomitting in pregnancyNausea and vomitting in pregnancy
Nausea and vomitting in pregnancy
 
how to write an abstract
how to write an abstracthow to write an abstract
how to write an abstract
 
Genetics for mrcog part1
Genetics for mrcog part1Genetics for mrcog part1
Genetics for mrcog part1
 
files
filesfiles
files
 
DKA
DKADKA
DKA
 
budget
budgetbudget
budget
 

Recently uploaded

MOTION MANAGEMANT IN LUNG SBRT BY DR KANHU CHARAN PATRO
MOTION MANAGEMANT IN LUNG SBRT BY DR KANHU CHARAN PATROMOTION MANAGEMANT IN LUNG SBRT BY DR KANHU CHARAN PATRO
MOTION MANAGEMANT IN LUNG SBRT BY DR KANHU CHARAN PATROKanhu Charan
 
👉 Guntur Call Girls Service Just Call 🍑👄7427069034 🍑👄 Top Class Call Girl Ser...
👉 Guntur Call Girls Service Just Call 🍑👄7427069034 🍑👄 Top Class Call Girl Ser...👉 Guntur Call Girls Service Just Call 🍑👄7427069034 🍑👄 Top Class Call Girl Ser...
👉 Guntur Call Girls Service Just Call 🍑👄7427069034 🍑👄 Top Class Call Girl Ser...chaddageeta79
 
Call Girls Mussoorie Just Call 8854095900 Top Class Call Girl Service Available
Call Girls Mussoorie Just Call 8854095900 Top Class Call Girl Service AvailableCall Girls Mussoorie Just Call 8854095900 Top Class Call Girl Service Available
Call Girls Mussoorie Just Call 8854095900 Top Class Call Girl Service AvailableJanvi Singh
 
Female Call Girls Sawai Madhopur Just Call Dipal 🥰8250077686🥰 Top Class Call ...
Female Call Girls Sawai Madhopur Just Call Dipal 🥰8250077686🥰 Top Class Call ...Female Call Girls Sawai Madhopur Just Call Dipal 🥰8250077686🥰 Top Class Call ...
Female Call Girls Sawai Madhopur Just Call Dipal 🥰8250077686🥰 Top Class Call ...Dipal Arora
 
Female Call Girls Tonk Just Call Dipal 🥰8250077686🥰 Top Class Call Girl Serv...
Female Call Girls Tonk  Just Call Dipal 🥰8250077686🥰 Top Class Call Girl Serv...Female Call Girls Tonk  Just Call Dipal 🥰8250077686🥰 Top Class Call Girl Serv...
Female Call Girls Tonk Just Call Dipal 🥰8250077686🥰 Top Class Call Girl Serv...Dipal Arora
 
ANATOMY AND PHYSIOLOGY OF REPRODUCTIVE SYSTEM.pptx
ANATOMY AND PHYSIOLOGY OF REPRODUCTIVE SYSTEM.pptxANATOMY AND PHYSIOLOGY OF REPRODUCTIVE SYSTEM.pptx
ANATOMY AND PHYSIOLOGY OF REPRODUCTIVE SYSTEM.pptxSwetaba Besh
 
Shazia Iqbal 2024 - Bioorganic Chemistry.pdf
Shazia Iqbal 2024 - Bioorganic Chemistry.pdfShazia Iqbal 2024 - Bioorganic Chemistry.pdf
Shazia Iqbal 2024 - Bioorganic Chemistry.pdfTrustlife
 
ANATOMY AND PHYSIOLOGY OF RESPIRATORY SYSTEM.pptx
ANATOMY AND PHYSIOLOGY OF RESPIRATORY SYSTEM.pptxANATOMY AND PHYSIOLOGY OF RESPIRATORY SYSTEM.pptx
ANATOMY AND PHYSIOLOGY OF RESPIRATORY SYSTEM.pptxSwetaba Besh
 
Female Call Girls Sikar Just Call Dipal 🥰8250077686🥰 Top Class Call Girl Serv...
Female Call Girls Sikar Just Call Dipal 🥰8250077686🥰 Top Class Call Girl Serv...Female Call Girls Sikar Just Call Dipal 🥰8250077686🥰 Top Class Call Girl Serv...
Female Call Girls Sikar Just Call Dipal 🥰8250077686🥰 Top Class Call Girl Serv...Dipal Arora
 
Face and Muscles of facial expression.pptx
Face and Muscles of facial expression.pptxFace and Muscles of facial expression.pptx
Face and Muscles of facial expression.pptxDr. Rabia Inam Gandapore
 
💞Call Girls Agra Just Call 🍑👄9084454195 🍑👄 Top Class Call Girl Service Agra A...
💞Call Girls Agra Just Call 🍑👄9084454195 🍑👄 Top Class Call Girl Service Agra A...💞Call Girls Agra Just Call 🍑👄9084454195 🍑👄 Top Class Call Girl Service Agra A...
💞Call Girls Agra Just Call 🍑👄9084454195 🍑👄 Top Class Call Girl Service Agra A...Inaayaeventcompany
 
Physicochemical properties (descriptors) in QSAR.pdf
Physicochemical properties (descriptors) in QSAR.pdfPhysicochemical properties (descriptors) in QSAR.pdf
Physicochemical properties (descriptors) in QSAR.pdfRAJ K. MAURYA
 
Call Now ☎ 9549551166 || Call Girls in Dehradun Escort Service Dehradun
Call Now ☎ 9549551166  || Call Girls in Dehradun Escort Service DehradunCall Now ☎ 9549551166  || Call Girls in Dehradun Escort Service Dehradun
Call Now ☎ 9549551166 || Call Girls in Dehradun Escort Service DehradunJanvi Singh
 
👉 Gulbarga Call Girls Service Just Call 🍑👄7427069034 🍑👄 Top Class Call Girl S...
👉 Gulbarga Call Girls Service Just Call 🍑👄7427069034 🍑👄 Top Class Call Girl S...👉 Gulbarga Call Girls Service Just Call 🍑👄7427069034 🍑👄 Top Class Call Girl S...
👉 Gulbarga Call Girls Service Just Call 🍑👄7427069034 🍑👄 Top Class Call Girl S...chaddageeta79
 
HISTORY, CONCEPT AND ITS IMPORTANCE IN DRUG DEVELOPMENT.pptx
HISTORY, CONCEPT AND ITS IMPORTANCE IN DRUG DEVELOPMENT.pptxHISTORY, CONCEPT AND ITS IMPORTANCE IN DRUG DEVELOPMENT.pptx
HISTORY, CONCEPT AND ITS IMPORTANCE IN DRUG DEVELOPMENT.pptxDhanashri Prakash Sonavane
 
The Clean Living Project Episode 23 - Journaling
The Clean Living Project Episode 23 - JournalingThe Clean Living Project Episode 23 - Journaling
The Clean Living Project Episode 23 - JournalingThe Clean Living Project
 
👉 Saharanpur Call Girls Service Just Call 🍑👄7427069034 🍑👄 Top Class Call Girl...
👉 Saharanpur Call Girls Service Just Call 🍑👄7427069034 🍑👄 Top Class Call Girl...👉 Saharanpur Call Girls Service Just Call 🍑👄7427069034 🍑👄 Top Class Call Girl...
👉 Saharanpur Call Girls Service Just Call 🍑👄7427069034 🍑👄 Top Class Call Girl...chaddageeta79
 
Cardiac Output, Venous Return, and Their Regulation
Cardiac Output, Venous Return, and Their RegulationCardiac Output, Venous Return, and Their Regulation
Cardiac Output, Venous Return, and Their RegulationMedicoseAcademics
 
Difference Between Skeletal Smooth and Cardiac Muscles
Difference Between Skeletal Smooth and Cardiac MusclesDifference Between Skeletal Smooth and Cardiac Muscles
Difference Between Skeletal Smooth and Cardiac MusclesMedicoseAcademics
 
VIP ℂall Girls Arekere Bangalore 6378878445 WhatsApp: Me All Time Serviℂe Ava...
VIP ℂall Girls Arekere Bangalore 6378878445 WhatsApp: Me All Time Serviℂe Ava...VIP ℂall Girls Arekere Bangalore 6378878445 WhatsApp: Me All Time Serviℂe Ava...
VIP ℂall Girls Arekere Bangalore 6378878445 WhatsApp: Me All Time Serviℂe Ava...deepakkumar115120
 

Recently uploaded (20)

MOTION MANAGEMANT IN LUNG SBRT BY DR KANHU CHARAN PATRO
MOTION MANAGEMANT IN LUNG SBRT BY DR KANHU CHARAN PATROMOTION MANAGEMANT IN LUNG SBRT BY DR KANHU CHARAN PATRO
MOTION MANAGEMANT IN LUNG SBRT BY DR KANHU CHARAN PATRO
 
👉 Guntur Call Girls Service Just Call 🍑👄7427069034 🍑👄 Top Class Call Girl Ser...
👉 Guntur Call Girls Service Just Call 🍑👄7427069034 🍑👄 Top Class Call Girl Ser...👉 Guntur Call Girls Service Just Call 🍑👄7427069034 🍑👄 Top Class Call Girl Ser...
👉 Guntur Call Girls Service Just Call 🍑👄7427069034 🍑👄 Top Class Call Girl Ser...
 
Call Girls Mussoorie Just Call 8854095900 Top Class Call Girl Service Available
Call Girls Mussoorie Just Call 8854095900 Top Class Call Girl Service AvailableCall Girls Mussoorie Just Call 8854095900 Top Class Call Girl Service Available
Call Girls Mussoorie Just Call 8854095900 Top Class Call Girl Service Available
 
Female Call Girls Sawai Madhopur Just Call Dipal 🥰8250077686🥰 Top Class Call ...
Female Call Girls Sawai Madhopur Just Call Dipal 🥰8250077686🥰 Top Class Call ...Female Call Girls Sawai Madhopur Just Call Dipal 🥰8250077686🥰 Top Class Call ...
Female Call Girls Sawai Madhopur Just Call Dipal 🥰8250077686🥰 Top Class Call ...
 
Female Call Girls Tonk Just Call Dipal 🥰8250077686🥰 Top Class Call Girl Serv...
Female Call Girls Tonk  Just Call Dipal 🥰8250077686🥰 Top Class Call Girl Serv...Female Call Girls Tonk  Just Call Dipal 🥰8250077686🥰 Top Class Call Girl Serv...
Female Call Girls Tonk Just Call Dipal 🥰8250077686🥰 Top Class Call Girl Serv...
 
ANATOMY AND PHYSIOLOGY OF REPRODUCTIVE SYSTEM.pptx
ANATOMY AND PHYSIOLOGY OF REPRODUCTIVE SYSTEM.pptxANATOMY AND PHYSIOLOGY OF REPRODUCTIVE SYSTEM.pptx
ANATOMY AND PHYSIOLOGY OF REPRODUCTIVE SYSTEM.pptx
 
Shazia Iqbal 2024 - Bioorganic Chemistry.pdf
Shazia Iqbal 2024 - Bioorganic Chemistry.pdfShazia Iqbal 2024 - Bioorganic Chemistry.pdf
Shazia Iqbal 2024 - Bioorganic Chemistry.pdf
 
ANATOMY AND PHYSIOLOGY OF RESPIRATORY SYSTEM.pptx
ANATOMY AND PHYSIOLOGY OF RESPIRATORY SYSTEM.pptxANATOMY AND PHYSIOLOGY OF RESPIRATORY SYSTEM.pptx
ANATOMY AND PHYSIOLOGY OF RESPIRATORY SYSTEM.pptx
 
Female Call Girls Sikar Just Call Dipal 🥰8250077686🥰 Top Class Call Girl Serv...
Female Call Girls Sikar Just Call Dipal 🥰8250077686🥰 Top Class Call Girl Serv...Female Call Girls Sikar Just Call Dipal 🥰8250077686🥰 Top Class Call Girl Serv...
Female Call Girls Sikar Just Call Dipal 🥰8250077686🥰 Top Class Call Girl Serv...
 
Face and Muscles of facial expression.pptx
Face and Muscles of facial expression.pptxFace and Muscles of facial expression.pptx
Face and Muscles of facial expression.pptx
 
💞Call Girls Agra Just Call 🍑👄9084454195 🍑👄 Top Class Call Girl Service Agra A...
💞Call Girls Agra Just Call 🍑👄9084454195 🍑👄 Top Class Call Girl Service Agra A...💞Call Girls Agra Just Call 🍑👄9084454195 🍑👄 Top Class Call Girl Service Agra A...
💞Call Girls Agra Just Call 🍑👄9084454195 🍑👄 Top Class Call Girl Service Agra A...
 
Physicochemical properties (descriptors) in QSAR.pdf
Physicochemical properties (descriptors) in QSAR.pdfPhysicochemical properties (descriptors) in QSAR.pdf
Physicochemical properties (descriptors) in QSAR.pdf
 
Call Now ☎ 9549551166 || Call Girls in Dehradun Escort Service Dehradun
Call Now ☎ 9549551166  || Call Girls in Dehradun Escort Service DehradunCall Now ☎ 9549551166  || Call Girls in Dehradun Escort Service Dehradun
Call Now ☎ 9549551166 || Call Girls in Dehradun Escort Service Dehradun
 
👉 Gulbarga Call Girls Service Just Call 🍑👄7427069034 🍑👄 Top Class Call Girl S...
👉 Gulbarga Call Girls Service Just Call 🍑👄7427069034 🍑👄 Top Class Call Girl S...👉 Gulbarga Call Girls Service Just Call 🍑👄7427069034 🍑👄 Top Class Call Girl S...
👉 Gulbarga Call Girls Service Just Call 🍑👄7427069034 🍑👄 Top Class Call Girl S...
 
HISTORY, CONCEPT AND ITS IMPORTANCE IN DRUG DEVELOPMENT.pptx
HISTORY, CONCEPT AND ITS IMPORTANCE IN DRUG DEVELOPMENT.pptxHISTORY, CONCEPT AND ITS IMPORTANCE IN DRUG DEVELOPMENT.pptx
HISTORY, CONCEPT AND ITS IMPORTANCE IN DRUG DEVELOPMENT.pptx
 
The Clean Living Project Episode 23 - Journaling
The Clean Living Project Episode 23 - JournalingThe Clean Living Project Episode 23 - Journaling
The Clean Living Project Episode 23 - Journaling
 
👉 Saharanpur Call Girls Service Just Call 🍑👄7427069034 🍑👄 Top Class Call Girl...
👉 Saharanpur Call Girls Service Just Call 🍑👄7427069034 🍑👄 Top Class Call Girl...👉 Saharanpur Call Girls Service Just Call 🍑👄7427069034 🍑👄 Top Class Call Girl...
👉 Saharanpur Call Girls Service Just Call 🍑👄7427069034 🍑👄 Top Class Call Girl...
 
Cardiac Output, Venous Return, and Their Regulation
Cardiac Output, Venous Return, and Their RegulationCardiac Output, Venous Return, and Their Regulation
Cardiac Output, Venous Return, and Their Regulation
 
Difference Between Skeletal Smooth and Cardiac Muscles
Difference Between Skeletal Smooth and Cardiac MusclesDifference Between Skeletal Smooth and Cardiac Muscles
Difference Between Skeletal Smooth and Cardiac Muscles
 
VIP ℂall Girls Arekere Bangalore 6378878445 WhatsApp: Me All Time Serviℂe Ava...
VIP ℂall Girls Arekere Bangalore 6378878445 WhatsApp: Me All Time Serviℂe Ava...VIP ℂall Girls Arekere Bangalore 6378878445 WhatsApp: Me All Time Serviℂe Ava...
VIP ℂall Girls Arekere Bangalore 6378878445 WhatsApp: Me All Time Serviℂe Ava...
 

Medscape articles

  • 1. A journey of a thousand miles begins with a single step | Yazid Jibrel Royal Medical Services OBGYN resident TWITTER@JIBREL MED STUDENTS AND RESIDENTS ARTICLES
  • 2. Table of Content 1 Table of Content Table of Content ...........................................................................................................................................1 When did you know? ....................................................................................................................................3 Do you have any suggestions on how to rank residency programs for the Match? ....................................6 When I apply for residency, whom should I ask to write a letter of recommendation? Are these letters really important? ..........................................................................................................................................8 Don’t Ask for a Letter of Recommendation… Ask for a Strong Letter of Recommendation!.....................10 How Can I Distinguish Myself on Clinical Rotations?..................................................................................11 I am getting ready to apply for residency. Who should I ask to write my letters of recommendation, and how should I approach them? ....................................................................................................................13 Get the Residency You Want: Tips and Tools: USMLE Scores, Matching Formulas, and More..................17 How Can I Find Love During Training? ........................................................................................................22 Every time I work with a new resident or attending, it seems like the requirements for my clinical case presentations change. Do you have any tips to help me predict how I should present? ..........................24 It is hard for me as a young student to interview and examine patients. I feel like I am intruding into their personal world without having a lot to offer them, given my limited knowledge base. Do you have any advice? ........................................................................................................................................................26 How Should I Deal With Gunners?..............................................................................................................28 Are there any advantages to working outside medicine before you begin medical training?...................30 Once I start practicing in the "real world," can I still change specialties if I feel that I made a mistake? Or will I be stuck in that career, no matter what?...........................................................................................31 I am starting my residency soon, and I am worried about being prepared. What resources should I bring to the hospital? How will I know what to do when I'm alone during night call? .......................................37 Twelve Steps for Choosing a Specialty........................................................................................................40 How Should I Choose a Medical Specialty? ................................................................................................42 I'm really worried that maybe I shouldn't be a doctor. What should I do?................................................44 Loving What You Do....................................................................................................................................46 Top 10 Reasons Why I Love My Job............................................................................................................48 How to be a Good Intern ............................................................................................................................50 Who Should Be a Doctor?...........................................................................................................................51 Developing Empathy in Future Physicians..................................................................................................52 In what order should I schedule my clinical rotations? Does it depend on the specialty I am choosing?.56 I would like to be a research assistant or pursue my own research project. How can I find a suitable professor to work with, and how should I approach one?.........................................................................58
  • 3. Table of Content 2 Taking Time Off Between Medical School and Residency..........................................................................60 Resume Design for Success.........................................................................................................................62 I'm thinking about doing an elective away from my school during fourth year. What do I need to do to arrange that? ..............................................................................................................................................64 Can You Learn How to Break Bad News?....................................................................................................67 Are Board Scores Important When Applying for Residency? .....................................................................70 How Can I Increase My Confidence in Doing Physical Exams?...................................................................72 I didn't enjoy my third year of medical school and now I'm not sure if I want to do a residency anymore. What should I do?.......................................................................................................................................74 How Can I Be Successful on the Job?..........................................................................................................76 How Do You Handle Pre-examination Stress?............................................................................................78 What If I Match a Residency Program I Don't Want?.................................................................................80 Why Doesn't My Resident Trust Me? .........................................................................................................82 What is more important in practicing medicine, the theoretical knowledge of the science or the practical intricacies and skills?...................................................................................................................................84 How can I remember and integrate the basic subjects of medical school with real clinical practice? ......86 I just finished medical school and am now an intern. How can I already be expected to teach medical students?.....................................................................................................................................................88 How Can I Predict My Success in the Residency Match?............................................................................90 How Can I Improve My Surgical Skills? .......................................................................................................92 How Can I Find a Mentor? ..........................................................................................................................94 How Should I Choose a Residency Program?..............................................................................................96 What Is the Best Way to Learn Procedures? Doing Them..........................................................................98 Do I Need To Do Research?.......................................................................................................................102 You will Survive.........................................................................................................................................103 Spicing up medical education ...................................................................................................................107 My First Day as a Doctor...........................................................................................................................112 Writing A Medical Case Report.................................................................................................................117 PDAs and Smartphones: Clinical Tools for Physicians...............................................................................120
  • 4. When did you know? 3 When did you know? Joshua Batt, Medical Student, Emergency Medicine, 10:19PM Nov 6, 2010 As I entered the gross anatomy lab within my first week of classes -- white-washed walls, stainless steel dissection tables, and fresh white lab coats moving about the room -- I knew I was in medical school. The feeling of making progress hit me when I saw the sea of cleanliness, the closest thing to a sterile environment and the resting place of our first patients. That feeling was not to be the last. The next significant affirmation was getting "pimped" on my first rotation. I knew then that I was beginning a career of integrative thinking seasoned with a little medical Trivial Pursuit. Although not as positive as the former experience, getting the question wrong was exactly the introduction I expected from this caliber of education; no matter, I'm in medical school, right where I want to be. Such defining moments may have a make or break effect. Whether rocking a board exam or losing a patient, colorful experiences paint our character and how we will one day practice medicine. These seem to be the very same milestones that pave our way to graduation. But the first is where the marathon begins. I have often wondered what other students define as landmark experiences of entry into their profession. Is it the endless reading assignments, the account-emptying tuition costs, suturing a laceration or overwhelming responsibility of caring for the life of another individual? Do you remember the feeling or event that made you feel like a soon-to-be-doctor or a physician? I would like to know... when did you know?
  • 5. A 12-step Program for Step 1? 4 A 12-step Program for Step 1? Rosalyn Plotzker, Medical Student, Family Medicine, 04:23PM Jul 16, 2010 Hello from the No Man's land, between the boards and third year. I just got back from post-USMLE rehab. While I was studying, I found unlimited advice about Step 1 preparation. Mysterious experts with personal websites weighed in on everything -- what to do early second year (buy First AID), the weeks before the test (do practice questions until you go blind), up until Step 1 Eve when you're packing your lunch and resorting to prayer ("just stay calm"... right). But where were the experts afterward? How do people recover from the inevitable post traumatic stress? Should you sprint from the Prometric center to the nearest happy hour? Seclude yourself in your living room and watch Transformers movies? Get on the first boat to the Bahamas? You can take the med student out of med school. But how much sunburn and bad acting does it take to get the school out of the student? My plan was to flee the country. Then, one month before the test, my vacation plans fell through. I was left with an empty week, and not enough student loan money to finance a trip to, well, anywhere. I needed something to distract me, a place totally strange, preferably beautiful, and cheap. A month later I was at the Tuscon international airport. I signed up to volunteer in the desert. Like a lot of students, I'm a sucker for volunteer work. First of all, it's free (or there's a small donation, way less than a week's vacation). Second, there are no decisions. Why fumble through a guidebook? Third, other volunteers tend to be good company. No risk of a beach full of whiney high maintenance tourists. Besides, I had my personal reasons for volunteering. I wanted to use my body to work, rather than my mind. I wanted to be with people who did not know me as a med student. Lately there had been moments when I wondered, "Has medical school taken out my soul and replaced it with a pathology textbook?" Maybe being with strangers could answer that. And, like most med students, I wanted the heat to evaporate memories of Goljian Lectures, First AID, Kaplan books, USMLE World, BRS - I'll stop there. A friend told me about No More Deaths, which is a non-profit that provides medical aid to migrants crossing through Sonora, especially people who are severely dehydrated or injured. As part of the desert team, I hiked through trails, looking for people who needed help. We carried food, water, and medical supplies in our backpacks. The boiling point for Step 1 is 108 degrees Fahrenheit. That's about when it evaporates. My team was wonderful. The landscape was unreal. But, by the third day my body was as exhausted as my mind. And, rather than hide my career path, I blurted out obscure medical facts at any opportunity. "No, I haven't seen any scorpions around my tent. Hey, did you know that scorpion bites cause pancreatitis?" So much for taking the student out of school. I couldn't stop. Keep reading. The silver linings are much more impressive than this story's clouds.
  • 6. A 12-step Program for Step 1? 5 Obviously, this group deals with a sensitive political issue, especially given the controversy over SB1070 right now in Arizona - not to mention the general attitudes towards immigration in the southwest. My personal feelings are that it's a good idea to help migrants if they need it, because it's a good thing to help anyone if you can. I'll leave the politics there for now. I will add that I was originally drawn to medicine because of health care activism work that I did in college. When I decided to be a physician, I imagined myself working in third world countries, volunteering for Doctors without Borders, or maybe starting my own clinic. (Who didn't?) I hardly had time for any of it in med school. There was barely time to get through the coursework. I treated multiple choice questions, not patients. No wonder the soulectomy- textbook-transplant. But, while I was in Arizona, I remembered the potent excitement of my career: the rush of treating someone who wouldn’t have been okay otherwise. The thrill of trying to heal someone. It knocked the Step 1 right out of me. Someday I will write a personal website on how to recover from the boards. It will be a bright yellow background, and in bold black letters it will say: Why did you apply to med school? Okay. So, now go do that.
  • 7. Do you have any suggestions on how to rank residency programs for the Match? 6 Do you have any suggestions on how to rank residency programs for the Match? Response from Daniel J. Egan, MD Associate Attending Physician, Department of Emergency Medicine, St. Luke's-Roosevelt Hospital Center, New York, NY The other day I was interviewing an applicant for a position in our residency program. She told me, "I think that I know who I want to rank at the top of my list." (This program, of course!). "It's the middle of my list that I'm having trouble with. Do you have any suggestions?" It's that time of year. All of you fourth-year medical students are wrapping up or have completed your interview journey. You may have traveled all over the country. You certainly spent a small fortune for airfare and hotels. Hopefully you accumulated some frequent-flier miles in the process. You also likely have sat through 10-15 sessions describing programs, listened to residency directors try to convince you that their program is the best, and had one too many "night before" social events with the residents in each program. So how do you make your list? Is it clear to you which program should be first? Is it less clear to you, like my medical student, what to do with the middle of your list? The most important thing for you to take home from this process is that your rank list should be truly that. You should rank programs in the order in which you would like to match. Many people believe that there is a way to outsmart the algorithm. Or maybe one program has given you feedback that makes you think they really want you. At the end of the day, you are dealing with perhaps the most formidable time in your development as a physician, and you should go where you want to be. So the list should be your own. It should not be the order that your dean or faculty advisor says it should be (they have their own interests in mind as well) but the rank order that would make you most happy. There are a number of factors to consider as you think about each program. The first and foremost is something that cannot be determined by a formula or list of pros and cons. The question is, Can you see yourself as a resident in that program? As you have gone through the process, you likely spent a day somewhere where you thought there was no way you would fit in. Additionally, there were likely others that left you almost giddy with the feeling that it would be a perfect fit. I still recall the chairman of the program where I trained meeting with us on the interview day. He casually spoke about a gut feeling of whether or not the glove fits. There is something important about your gut. Frankly, it is something that will get you through many difficult decisions in medicine, and choosing a residency is certainly one of those. Certain criteria should always be considered when evaluating a residency program: patient population, tertiary care exposure, clinic experience for the specialties with outpatient components, resident independence in regard to decision-making, electives, and where graduates go for fellowships or jobs. Other criteria that should play a role in your decision-making include:
  • 8. Do you have any suggestions on how to rank residency programs for the Match? 7 Location. This is not insignificant. There are many glamorous cities in the country, but not every city is made for every person. Do you know anyone in the city that you are considering? Will you have any kind of support system outside of the hospital? You will immediately establish lifelong friendships during residency, and there is something exciting about starting over in a new place. But everyone has a different personality, and some people find it easier if a network already exists during a very challenging time. Housing and salary. Is housing provided? If not, will the program's salary allow you to rent an apartment and still have money left over for your usual expenses? (See our previous columns on the accrual of credit card debt and how this should be avoided at all costs!) Are the residents happy? You will hear rumors along the interview trail. Pay attention. Obviously, rumors are rumors, but hearing that residents in a particular program are very happy is an important piece of information. Do you know any graduates from your medical school who are there? We all have a loyalty to our school. We want the best for students from our school. Often, these residents will be fantastic resources as you try to get real, honest information. What is the academic vigor of the program? If you have a long-term goal related to academics, you want a program that will help foster that. If faculty members are not doing research and publishing, it may be more difficult for you to find a mentor. What are the daily or weekly conferences like? What are the academic requirements of the residents? Teaching environment. With shorter work hours and increased patient volumes affecting all programs, many residents feel that the clinical teaching suffers. Ask about bedside teaching. The accrediting agency mandates a core content of lectures and conferences, but it is really the day-to-day operations that help you develop your clinical skills. You may best experience this with a second look. Show up on a noninterview day. See how the residents interact with each other and their attending staff. Finally, I'll share some advice that I got from my med school dean. He told us to put Post-It notes around our apartments with each program listed in order. Put them somewhere that you will see every day, preferably multiple times. Believe it or not, that gut will start talking again. Imagine yourself opening the envelope with each program named inside, and likely you will have a different response to each one. Looking at this list and thinking about that feeling may help you tweak the order. It certainly helped me solidify my final list, as I grew more excited about the prospect of my first choice. And remember, the formula should work in favor of you, the student. Don't try to play a game. Do not shorten your list because you think you are guaranteed a spot based on program feedback. Take a phone call or email from a program with a grain of salt. Do what YOU want to do with your list!
  • 9. When I apply for residency, whom should I ask to write a letter of recommendation? Are these letters really important? 8 When I apply for residency, whom should I ask to write a letter of recommendation? Are these letters really important? Response from Daniel J. Egan, MD Associate Attending Physician, Department of Emergency Medicine, St. Luke's-Roosevelt Hospital Center, New York, NY As someone who routinely reviews applications to our residency program, I can tell you that letters of recommendation are very important to a successful match. The initial screening process typically involves board scores, rotation grades, medical school grades, and Dean's letter summary statement. However, after an applicant has made that cut, letters of recommendation play an important role in the evaluation process and truly give insight beyond the standardization in the rest of the application. Several approaches to letters of recommendation can be taken. The most obvious person to ask for a letter is a mentor who knows you well. If you have worked with a mentor on a research project or have formed a relationship outside of a rotation or the classroom, this person should write a letter on your behalf. Most likely, his or her letter will provide more personal insight about you as an individual and capture the reader's (i.e., the interviewer's) attention. I often pull interview questions from these letters to ask about the mentorship or project on which a student worked. The second most obvious “letter writer” for you should be someone involved in a position of leadership in the clerkship of your specialty choice. Hopefully you have done well in the clerkship of your future specialty, and you will need to have someone from the faculty speak on your behalf. If you do not include a letter from someone in your specialty, it will be a glaring red flag to those of us reading through your application. In fact, some specialties have requirements about a certain number of letters from staff in your specialty. Be sure to find this out. Additionally, some specialties (such as emergency medicine) require a specific form letter; obviously, it is important to make sure you follow these guidelines. The question becomes, Who should write your letter within a given rotation? A letter from a resident is not an option. A brand-new attending in his or her first year out of residency may also not be the ideal candidate. It may be to your advantage to approach someone who is more senior within the department. Ultimately, the content of the letter is important, but supportive statements from a chair, associate or full professor, or residency director will be taken seriously. Additionally, many of these faculty are nationally known, and the reader next to me may know them personally or by name, giving more credence to the letter. Faculty members are frequently asked to write letters. People who routinely write letters include the medical student director for a rotation and members of the residency leadership. Those of us reading a letter look for certain phrases and comments. If a faculty member is not familiar with letter writing, he or she may not include these statements and it may be hard for an interview committee to interpret the letter. A medical student clerkship director will also probably include statements or at least a summary from multiple faculty members and give a consensus opinion. One individual faculty member can only
  • 10. When I apply for residency, whom should I ask to write a letter of recommendation? Are these letters really important? 9 speak for himself or herself. Another tip: if you are unsure of a person's opinion of you, avoid asking him or her to write a letter. Clearly, you do not want a “bad” or even mediocre letter in your file. Some tips for deciding whom to ask:  Speak to upperclassmen who have completed the rotation. Some faculty members in your school have letter-writing reputations.  Consider your approach to the faculty member. I have been asked explicitly, “Do you think you would be able to write me a strong letter of support?” Many faculty will tell you that they are not comfortable writing a letter even without this specific question, but it is always hard to say "no" to someone. However, if you deliver this more directed question using the adjective "strong," it gives someone an out to say "no" if they do not think they can write you an adequate letter.  Ask the residency director at your school for advice. Tell the residency director whom you are considering asking to write letters for you. He or she may steer you in another direction if that faculty member may not be the best option.  Speak openly with your Dean of Student Affairs. The medical school is acting in your best interest, and the leadership wants students to be successful and match at competitive programs. One last piece of advice: Ask for your letter as soon as possible after your clerkship, when you are still fresh in the mind of the faculty member. Also, remember that every letter you collect does not ultimately need to be included in your application. Better to have more than you need than not enough.
  • 11. Don’t Ask for a Letter of Recommendation… Ask for a Strong Letter of Recommendation! 10 Rules of the Road Don’t Ask for a Letter of Recommendation… Ask for a Strong Letter of Recommendation! Joel Schofer, MD, Emergency Medicine, 08:20AM Aug 11, 2010 Students often approach their faculty and teachers and ask, "Can you write me a letter of recommendation?" The literal answer to this question is always yes. They "can" write you a letter of recommendation. Anyone can. The implied meaning of this question is, "Can you write me a STRONG letter of recommendation?" No one wants a letter of recommendation that is anything but strong. A mediocre or poor letter of recommendation can be the kiss of death to any application, but, sadly, unless you ask about the strength of their letter, a less than stellar letter is what you may wind up with. And because of the privacy of most letters of recommendation, you may never even find out about it! I encourage all students to ask their faculty for exactly what they want, asking, "Can you write me a strong letter of recommendation?" (or some other similar variant). You may be surprised by the answer to this question. Some students will be shocked when a supportive faculty indicates that they could write a letter, but that it wouldn't be all that supportive. It is much better to know this, however, before a weak letter is written and submitted on your behalf. In summary, don't ask for a letter, ask for a STRONG letter. If you can't get one, find someone else to write you the letter.
  • 12. How Can I Distinguish Myself on Clinical Rotations? 11 How Can I Distinguish Myself on Clinical Rotations? Response from Graham Walker, MD Resident, Emergency Medicine, St. Luke's-Roosevelt Hospital Center, New York, NY Here are 10 tips to help you earn recognition as an outstanding member of the clinical team: 1. Be enthusiastic. Energy is contagious, and we all need more of it in medicine. Bring it to your rotations! Even if you're not interested in, say, surgery, look for a particular aspect that you do find interesting. Help out where you can. Volunteer to do more. It goes a long way. 2. Avoid complaining. As hard as you're working (and I'm sure you are!), your residents are working even harder (or at least have more responsibilities and are balancing more spinning plates). It's fine to join in when people are venting about how big your team's census is getting, but you won't be winning any points by saying how rough your job is. (We all know scut sucks, but if you don't do it, your resident will be doing it for you.) 3. Anticipate tasks that need to be done. Once you've spent a week or two on a service, you can start to see some patterns: calling for a nutrition consult, getting social work involved, collecting a patient's list of home medications, and requesting the old chart. Surprise your resident by thinking ahead, and you'll be at the head of the pack. 4. Take ownership of your patients. The point of clerkships is to prepare you to be a good clinician and a good resident. Write a good history and physical ; get your notes ready for rounds; and be the first one to know about any changes, such as a consultant's recommendations or the medications added while your team was post-call. 5. Help your intern or resident stay informed. Write good patient notes and provide frequent updates. Your residents are ultimately responsible, so let them know when laboratory results are back or if the radiologist just called with CT results, for example. 6. Ask questions. This is huge! It's your responsibility to learn patient management, so if you're not sure why your team is ordering a test or giving a certain medicine, ask. It will show your interest and allow your residents to do some teaching (and for most of us, it's enjoyable to pass on knowledge to someone at a stage of training). Just make sure to raise your questions at the right time: Asking to see the ligament of Treitz while your resident is trying to get control of a bleeding surgical field will not win you any points. 7. Don't lie. If you don't know the answer to a question, go ahead and say, "I don't know, but I will find out." This is especially true if you're asked about patient-related information; if you forgot to listen to the lungs, just admit it. It's much better than saying they were normal and then finding out the patient has rales and has been fluid overloaded all night. 8. Be nice to nurses and other staff. This should be as obvious as the Golden Rule, but sometimes students can overlook it. Be the first to introduce yourself, say "please" and "thank you," and try not to leave a mess. Behind any excellent doctor is a bunch of excellent nurses, no doubt.
  • 13. How Can I Distinguish Myself on Clinical Rotations? 12 9. Show up early. Or at least be on time. (See "Avoid complaining," above.) Your interns and residents on the team are getting even less sleep than you, and they are still making it in! 10. Come up with your own patient assessment and treatment plan. There's no better way (besides maybe #6, above) to show your colleagues that you're thinking and learning. You'll sometimes be wrong, but you're supposed to be wrong, right? Otherwise there'd be no reason to go through medical school and residency.
  • 14. I am getting ready to apply for residency. Who should I ask to write my letters of recommendation, and how should I approach them? 13 I am getting ready to apply for residency. Who should I ask to write my letters of recommendation, and how should I approach them? Response from Geoffrey Talmon, MD Surgical Pathology Fellow, Department of Laboratory Medicine and Pathology, Mayo Clinic, Rochester, Minnesota As a medical student applying for residency, I found it very uncomfortable to solicit letters of recommendation (LORs). After all, I needed letters from professor-level attendings who had the potential to affect my future. On top of that, I had heard that I was not supposed to review the letters before they were submitted. Questions percolated through my brain, ratcheting up my stress: Who, and how, should I ask? Would they say "yes"? Did they know me well enough to recommend me? What do these letters even say? It was not until I was involved as a resident in recruiting other house officers that I truly understood how LORs are used by programs and what constitutes a good one. In several ways, the residency Match process mirrors the job application process in the business world. LORs can be viewed as professional references. As such, they should discuss attributes that are not explicitly outlined in other application materials. Qualities, such as work ethic, willingness to learn, enthusiasm, and teamwork skills, give the audience (ie, interviewers and program directors) insight into how you would function as a resident in their program. In keeping with this purpose, the individuals who you ask should have worked with you enough to comment on these points, hopefully in a positive manner. To carry the most weight, this person should be a faculty member or practicing physician, not a resident. This can prove to be more difficult in certain specialties. On an internal medicine service, a medical student may work with the same team for an entire rotation. In my field of pathology, however, I worked with no fewer than 10 attendings during my 1-month rotation. I ended up having to ask for recommendations from those staff members who I believed were the best teachers and who saw glimpses of my enthusiasm. Asking an attending physician to write an LOR can be a daunting experience, especially if it is a person with whom you have not worked extensively, as is often the case with a department chair. Keep in mind that most physicians who regularly deal with medical students have experience in writing LORs and understand what is required. They often will ask for the perspectives of your supervising residents and other faculty to garner the required information. In fact, you can explicitly suggest this to the person writing your letter. In reviewing applications, I have seen writers mention that their letter represents a sort of "consensus opinion," which usually makes that letter at least as strong as any other. It is most effective to approach your potential writers during a time when you are not on their service; the earlier in your application process, the better. I found that scheduling an appointment with the staff physician's secretary was the most practical approach. Having a dedicated time gives you the opportunity to discuss your goals and plans while refreshing their memory about your time working with them. At the very least, you should provide a copy of your curriculum vitae and personal statement.
  • 15. I am getting ready to apply for residency. Who should I ask to write my letters of recommendation, and how should I approach them? 14 The best LORs that I have encountered integrate specific observations from the writer with information in the applicant's personal statement. (For example: "On a daily basis I saw the dedication to patients' spiritual needs that Ms. Smith mentioned in her personal statement as being an important part of her life.") The more information that you can provide (conversations, application materials, etc), the easier it will be for your writer to write a strong LOR. Finally, agree upon a "due date" for the letter, preferably about a month in the future. Resist the urge to review an LOR before it is submitted; although it may be difficult to accept that this portion of your application is outside your control, you should realize that residency programs view this "blindedness" as a control for bias. If you are overly concerned about what someone might write, you may want to reconsider asking that person to provide a recommendation. Another important aspect is ensuring that your letters are submitted on time. The writers likely are incredibly busy individuals, and they may need to be reminded of the deadline; at the same time, you do not wish to be a pest. If you see that your LOR has not been submitted by the agreed upon date, send a polite email asking whether the writer needs more information, and then reiterate the due date. This is where giving your writer ample time is helpful. In summary, the LOR is a professional reference that is meant to provide residency programs with additional information to determine whether you will fit in with their team. Strategies, such as setting up an appointment to converse with your potential writers, providing them with as much information as possible, and giving them plenty of lead time, can help them write an effective letter that ultimately benefits you the most. Quoted from Dr. Louay (Sam’s Friend) I think this is an important question because in this day and age the LOR are really what make or break you. Since USMLE scores are no longer reliable to differentiate people (every one scores high), US-written letters are what will carry you. I will try to break my answer into 3 categories to cover everything you asked. Content: 1. LOR has to be “excellent” otherwise it is damaging. After all, you chose which ones to send so everyone expects them to be at least very good. A half hearted letter is disastrous. 2. The length of the letter is less important than its content. A “great” letter of 1 paragraph is better than a “good” letter that is a 2-pager. Of course a long great letter is best. 3. The best letter for an FMG is the one that reports outstanding knowledge (better than our residents/students), being able to fit in the US system and carry patient load, excellent communication skills (includes language), and excellent rapport with the staff. If there is some “cool” personal info then it is even better.
  • 16. I am getting ready to apply for residency. Who should I ask to write my letters of recommendation, and how should I approach them? 15 4. It is near impossible to discover that a US letter is pre-set (cut and paste) unless the writer made mistakes. Sometimes you find in the middle of the letter a mention of a name other than the applicant’s. Or sometimes you find “he” instead of “she”! 5. The shorter the time of contact the less valuable the letter. 6. “Although” can be good or bad base on the context: -“Although he has an accent, his communication skills r great” is 50-50 - “In spite of different culture and language, he was able to deal with pt well.” is also 50-50. - “Although the medical system is different the one in Syria, he was able to deal with it like our students did” is excellent! -“Although he had some difficulties with English at the beginning, he did very well and improved throughout the month” is very bad! Source: 1. The most important factor of the letter is the writer. After all you are taking someone’s word, so he/she better be credible. Since most PDs will not know the writer, they look at the academic titles and leadership positions. Someone who is a professor is more credible than an assistant professor (older and wiser). Someone who is a department chief or a chairman of medicine is highly credible. Someone who deals with residents and students frequently (residency PD or a clerkship director) has credibility. 2. MD or DO does not make a difference. What makes a difference is academic rank/position. 3. American names are more credible than foreign names in general. A PD may wonder whether the “foreign” name is a relative or a family friend otherwise. This will be less of a factor if the rest of your “American” letters are consistent. 4. The institution is very important, the stronger the better. Excellent letters from the Mayo clinic or Duke are 10 times more important than excellent letters for UTMB (if you are better than their students then it is good to know what students are they comparing you to?). 5. LOR from outside the US are almost useless unless the program is NOT used to FMGs. Programs who see FMGs regularly know that these are usually generic letters that have no real meaning. Programs that seldom see a FMG may take them seriously for lack of experience with them. 6. Programs may call your letter writers if they are having a tough time making a decision how high to rank you. If they take the trouble of doing so, at least it means they are interested. General Strategy: 1. The more US letters the better (as long as they are very good). The Dean’s letter is enough from your school.
  • 17. I am getting ready to apply for residency. Who should I ask to write my letters of recommendation, and how should I approach them? 16 2. Inpatient medicine rotations are stronger than elective consultation. Observer as the least significant. There is no difference in regards to type of elective (except if you have an elective where you mostly watch, such as cath lab, then it is no good). Inpatient is generally better than outpatient because it is more likely to be rigorous. 3. In general, send the strongest letters (10/10 or 9/10) better than 7/10 no matter what they cover, as long as the writers are of the same caliber. A 8/10 from a famous tough guy is better than 10/10 from a young assistant professor. I hope this covers the topic adequately. Feel free to post follow up questions if you have any.
  • 18. Get the Residency You Want: Tips and Tools: USMLE Scores, Matching Formulas, and More 17 Get the Residency You Want: Tips and Tools: USMLE Scores, Matching Formulas, and More Introduction Medical students constantly hear that the fourth year is the best year of medical school. That may be true, but for most students, fourth year also means figuring out how to navigate the residency match process, which can seem chaotic and mysterious, full of myths and speculation. Students basically want answers to some simple questions: What is a particular program like? How competitive am I for a given program? To how many programs should I apply? What are residency directors really looking for? And how can I really shine in an interview? Although there is no one answer to any of these questions, there are experts who can provide some guidance. At the recent annual meeting of the Association of American Medical Colleges (AAMC), leaders from 2 schools offered their ideas about how students can be successful residency applicants. The following tips represent a summary of the insights shared by Paul Jones, MD, Associate Provost of Student Affairs and Director of Pediatric Otolaryngology and Resident Education at Rush Medical College, Chicago, Illinois, and Angela Nuzzarello, MD, MHPE, Associate Dean for Student Programs and Professional Development and Assistant Professor in the Departments of Psychiatry and Medical Education at the Feinberg School of Medicine, Northwestern University, Chicago, Illinois. USMLE Scores, Matching Formulas, and More Characteristics of applicants in the 2005 residency match are outlined in "Charting Outcomes in the Match,"[1] a publication of the AAMC and the National Resident Matching Program (NRMP). The report includes average scores on the US Medical Licensing Examination (USMLE) for applicants in the 2005 match. For some specialties, median USMLE scores and the ranges of scores were quite broad, so students should use these data only as a gauge of their competitiveness, not as a reason to choose a particular specialty, Dr. Jones said. Although some residency programs do not look at applicants' scores on the Step 2 Clinical Knowledge exam (known as the Step 2 CK), a growing number of programs do want to review these scores and are putting greater emphasis on students' performance in this area, Dr. Jones said. Students should check with individual programs to see whether they require Step 2 CK scores before they interview with the program or complete their rank lists, he advised. Dr. Nuzzarello recommended that students find a research project in the specialty area that they want to pursue. Rather than just listing publications or job experience on a résumé, students should present research projects by explaining their particular roles in the project, as well as describing the skills they acquired. Thus, even without a long list of publications, students can convey their level of understanding and proficiency in laboratory skills. Each residency program has its own "formula" for selecting applicants, and program directors may or may not share this formula with students. However, students should focus on trying to be themselves, Dr. Jones and Dr. Nuzzarello agreed, so that both the interviewer and the applicant can see what the other is really like. Despite what many students believe, residency program leaders can be just as surprised on Match Day as the applicants; they really do not know who will be filling their residency slots, and they are
  • 19. Get the Residency You Want: Tips and Tools: USMLE Scores, Matching Formulas, and More 18 eager to impress applicants just as students hope to impress them. In the end, the match process seems to work, the speakers said, so students should trust the process. Dr. Jones reviewed studies that looked at how some program directors use data to rank residency candidates. One survey of program directors in orthopaedics found that the top 10 things they valued were (in diminishing order):  the student's completion of a rotation at the program director's institution;  performance on USMLE Part 1;  rank in medical school;  formality and politeness at the interview;  personal appearance;  performance on questions of ethics at the interview;  letter of recommendation from an orthopaedic surgeon;  membership in the American Orthopaedic Association;  medical school reputation; and  the dean's letter.[2] Dr. Nuzzarello identified red flags that dissuade residency directors:  Failure of USMLE Step 1  Failure of a core clerkship  Unexplained leave of absence  Formal disciplinary action  "Extra" requirements or "extra" time  Obtuse personal statements  Negative letters of recommendation  Falsified or exaggerated entries on the résumé If, for whatever reason, a student took time off during medical school (even for a research rotation), he or she should explain this fully, because some faculty will only notice that the student did not graduate in 4 years, Dr. Jones said. Any student with one of the above "red flags" in their application should be prepared to address it during an interview. Dr. Jones added that, in his experience, students who are unsuccessful in the residency match are unaware of how they measure up against other applicants.
  • 20. Get the Residency You Want: Tips and Tools: USMLE Scores, Matching Formulas, and More 19 Students who have a red flag, but who also realize it and address it in their interviews, seem to do better, he said. Interview Tips and Tools Dr. Nuzzarello provided an overview of what students need to know and do before they begin the residency interview process. The following lists summarize her recommendations. Interview tips: 1. Know what is unique about the program 2. Be aware of any recent changes in the program or department 3. Learn about the department's faculty and ongoing research projects 4. Use the Internet to learn about the program and hospital 5. Speak with alumni 6. Study the geography of the program, ie, remote clinical sites 7. Research the community What to bring to the interview: 1. Directions and contact phone numbers 2. Any information you have on the program 3. Multiple copies of your CV 4. Copies of your papers and published articles 5. Portfolio, pen, pencil 6. Small suitcase, if traveling 7. Any updates to the Medical Student Performance Evaluation (MSPE) or transcript 8. An interview checklist Tips for the interview: 1. Start with a handshake and make good eye contact. 2. Don't be afraid to smile. 3. Be aware of your body language (eye contact, position in chair). 4. Show enthusiasm for the program, no matter how interested you are.
  • 21. Get the Residency You Want: Tips and Tools: USMLE Scores, Matching Formulas, and More 20 5. Answer each question and then pause; know when to stop. 6. Pay attention to interviewers' cues. 7. Be wary of programs that do not allow you to interact with residents. Questions and topics to anticipate in the interview: 1. Tell me about yourself. 2. What would you do if... (ethical dilemmas). 3. What attracts you to this specialty? 4. Describe the most interesting case you've seen. 5. Why should we choose you over other candidates? 6. Where do you see yourself in 10 years? 7. What are the biggest issues facing physicians today? 8. What is your greatest weakness? Questions to ask the faculty (interesting questions will make you stand out, as faculty often hear the same questions all day): 1. Are any program changes anticipated? 2. What percentage of residency graduates have passed the specialty board exams on the first try? 3. Do you assist graduates in finding a job? 4. What do you feel are the strengths of this program? 5. What do your graduates do after residency? (what types of jobs, where they are located) 6. What is the balance of didactics and clinical work? 7. How much contact do the residents have with faculty? 8. Are there opportunities for research? (only if you are interested) 9. Are there nonclinical requirements (research, writing, administrative tasks)? 10. Is there elective time? Questions to ask residents: 1. What aspects of the program need improvement? 2. What is one thing that you would change about this program?
  • 22. Get the Residency You Want: Tips and Tools: USMLE Scores, Matching Formulas, and More 21 3. How much contact do the residents have with faculty? 4. What do residents generally do with elective time? 5. How are residents evaluated? 6. Are you receiving adequate feedback? 7. Are there international opportunities? Questions not to ask: 1. What is the salary? 2. What is the benefit package? (This should be in the information packet.) 3. How much vacation do I get? 4. When can I start moonlighting? Finally, always treat the residency coordinator well. Your interview starts with the first contact you have with the program, and this is often with the coordinator. Many program directors ask the coordinators what they thought about applicants, so it pays to be nice to them. Also, students should follow up interviews with thank-you notes. For interviews with multiple faculty members, a single thank-you note to the program director will suffice. In fact, this is an opportunity to remind the interviewer about something interesting about yourself. In summary, the presenters emphasized that this is a match process where both sides are trying to get to know each other. Students cannot predict what program directors are looking for, so they should follow the above recommendations but they should also try to be themselves. This way, programs can determine which candidates are the best fit for them.
  • 23. How Can I Find Love During Training? 22 How Can I Find Love During Training? Question Developing a romantic relationship seems very difficult during medical training. Is there anything I can do to improve my prospects? Megan Fix, MD Attending Physician, Maine Medical Center, Portland, Maine Although I wish it were not so, there is no "love potion number 9" to magically establish a relationship. Maintaining strong relationships with others, however, is vital to your well-being during training. Can you find someone during this time? Yes. How do you find someone? My advice is to start by looking inward and be honest and true to yourself. Once you build a good relationship with yourself, do the same with your friends and family, and when those are strong and healthy, you will best be able to find and hold onto a significant other. There are many books and articles that focus on the "medical marriage" and on nurturing an established relationship, but in the interest of space, this article focuses on finding love during medical training. First, it is so important to love yourself. It seems simple, but with the long hours, negativity, and grueling demands of training, it is hard to remember. Some of us look outside ourselves to fulfill what we need inside. We are smart, accomplished physicians but we still feel small. We look for good grades and good comments from superiors to make us feel better inside. This need for approval can spill over into our relationships. The fact is that you are good enough just being you. If you really believe that, then others will, too. If they do not agree, or try to put you down, then they are not worth your time. If you are happy with yourself first, you will have so much more to give and to gain when you are in a relationship with another. Once you learn to truly love yourself, the next step is to be honest about what you want and need. This is important in residency not only because you have limited amounts of free time, but also because it is the time when most of us are emotionally maturing. Is it important to share religious beliefs with your partner? Is it important to share outdoor interests with them? Is it important to share interests in politics? Whatever it is, give yourself the liberty to trust what you need. When you think of a golden weekend, what is it that you want to do? Run, write, paint, or sit in a cafe? Whatever it is, make sure that you nurture your core values. Next, share these core values with others. Start with friendships. If you value cycling, for instance, find others who do as well. Join a club. Your happiness in doing things that you love will show, and it will spill out to those around you. If you love music, find a live music spot and bring along a medical colleague. Not only will you strengthen your friendship, but perhaps you will find a potential significant other. Avoid going out just to "find someone," because it often is superficial. Spend time being you and doing the things that you enjoy, and kindred spirits will be nearby. In his handbook on residency, Dr. Lyle Victor writes, "a
  • 24. How Can I Find Love During Training? 23 happy marriage involves the definition of mutually agreed upon lifestyles and goals.[1] " Establish goals for yourself; share them with someone else; and they will be much easier to embrace in a long-term relationship. People often debate dating someone who is "in medicine" or "not in medicine." There are pros and cons to each, but no right answer. Some will say, "I only meet people at the hospital" in defense of the former. Others will say, "We always talk about medicine; I need someone who lives in the real world" to support the latter. My advice is, again, be yourself and find people who are in line with what you like -- no matter where they work. What about Internet dating? I am a big advocate of this for medical residents. Three of my fellow residents have found their partners online and are amazingly happy. Internet dating can reduce the time you have to spend searching for someone, and it can help you find like-minded people in your area. The only pitfall to avoid is getting too overwhelmed. Remember that it is okay to be selective with your time. Your priority should be first to yourself and your patients, then to your dating. Your time is precious, and if you are not connecting with someone, politely say "no, thank you." Once you do find someone, however, whether they are in medicine or not, it is important to communicate. Focus on the positive aspects of being a physician. Share your passions and remember why you went into medicine in the first place. There are many negative aspects of training, but try not to dwell on those when you are meeting someone. For example, if you had a difficult patient encounter before a date, feel free to share what bothered you but don't dwell on the medicine aspects. Emotions are common to all of us. Share your passions and joys. Finally, be honest about your schedule. It is hard for anyone to understand the hours we put in during medical training. Do not try to meet someone at 6:00 pm when you know that you may not be out until 6:45. Just as you nurtured your relationship with yourself, you must nurture your relationship with another person. Even when you come home and are tired, remember that a healthy relationship is a 2-way street: You give, and they give. Feel good about giving. In his article on the well-being of physician relationships, Dr. Michael Myers states that "all relationships require care, patience, and nurturing.[2] " Start by nurturing yourself, and that foundation will serve as fertile ground for nurturing your relationship with your partner. Medical training is a defined period of time: for most of us, somewhere between 7 and 11 years. That is a long time to be down on yourself for not having a relationship, or for being in the wrong one. So, do what you can; enjoy who you are; share with those around you; and be excited about the potential of true happiness with someone who shares the good parts of you.
  • 25. Every time I work with a new resident or attending, it seems like the requirements for my clinical case presentations change. Do you have any tips to help me predict how I should present? 24 Every time I work with a new resident or attending, it seems like the requirements for my clinical case presentations change. Do you have any tips to help me predict how I should present? Response from Thomas E. Robey, MD, PhD Resident, Emergency Medicine, Yale-New Haven Hospital, New Haven, Connecticut I've been mulling over my response to this great question for several weeks, only because there are so many potential answers but also a number of pitfalls. So when my new senior resident on an off-service floor month gathered our team before rounds this morning, the answer jumped out at me: Just ask! In about 5 minutes, I knew how to structure my presentations for the rest of the month. It's natural to be apprehensive about rounds, and the guessing game about oral presentations doesn't end when you graduate from medical school. As long as you are junior to another doctor (in effect for your foreseeable future), you will need to adjust your presentation to accommodate your supervisor. And isn't that true when you talk to anyone? You'd use different language, mannerisms, and sentence structures for outreach at a grade school vs when you defend your PhD dissertation, right? In the same way, you can expect to use different styles on surgery or medicine clerkships. That said, I had a tough time with rounds in my third year of medical school. This was usually to my detriment, as evaluation comments would come back to the tune of, "Thomas understands the clinical information but struggles to present it in a concise manner on rounds." Of course, one attending complimented me on my monologues as she chastised a senior resident trying to interrupt me. She said, "I find Thomas' presentations refreshing. They remind me of reading Faulkner." I'm not sure whether that was actually a compliment. But I digress. The only contact you may have with your attending is on rounds, so it's important to make sure you're giving the presentation that he or she wants to hear. When you start a clerkship, ask your resident what he or she expects and what the attending prefers. Be specific! Here are some concrete questions to ask your senior resident:  What do you expect from my clinical presentation?  Do I present my plan by problem or by system?  Am I shooting for 10 minutes, 3 minutes, or 3 sentences?  Should I expect interruptions?  What does Dr. Attending like and hate in a presentation?  Does the team generally read from notes?  How are my presentations going?  Could you meet me 15 minutes before rounds to go over my presentation? I found that last one the most useful. Usually interns are too busy to help out before rounds, and the senior resident is better situated to offer feedback. Are you still unsure of the general skeleton of your
  • 26. Every time I work with a new resident or attending, it seems like the requirements for my clinical case presentations change. Do you have any tips to help me predict how I should present? 25 presentation? At some point in medical school, you should have received coaching for this. If you need more, the UCSD School of Medicine has an excellent online guide for comprehensive oral case presentations. As a medical student, you are allowed to err on the side of too much information, and as your experience grows, your presentation will shrink. Soon enough, you'll be an expert yourself!
  • 27. It is hard for me as a young student to interview and examine patients. I feel like I am intruding into their personal world without having a lot to offer them, given my limited knowledge base. Do you have any advice? 26 Question It is hard for me as a young student to interview and examine patients. I feel like I am intruding into their personal world without having a lot to offer them, given my limited knowledge base. Do you have any advice? Response from Daniel J. Egan, MD Associate Attending Physician, Department of Emergency Medicine, St. Luke's-Roosevelt Hospital Center, New York, NY First of all, I commend you on recognizing a level of discomfort with what you are doing. In medicine, we tend to normalize things that would not be considered normal anywhere else. Early on in your training, you are more likely to recognize and be uncomfortable with what the rest of the world may consider "abnormal." There is a unique and relatively rapid transition that occurs as you become a medical student. Most schools have now introduced a clinical exposure to some degree early in training. In the setting where I work, first-year students shadow me and begin interviewing patients as part of the Fundamentals of Clinical Medicine course. Just a short time ago, these students were regular college students; suddenly, after putting on the white coat, they have taken on a very different role. Your statements raise several issues. The first is the basic difficulty in examining and interviewing patients. You are suddenly prying into the history (both previous and present) of these individuals. We frequently label patients by their diagnoses (eg, the "chest pain" in room 2; "stroke" in room 7). However, behind each diagnosis is a person with a medical history, a social history, and family members who care about them. You are expected to ask questions about all of those things, and in return, you have an expectation that the patient will share his or her answers with you. But what if the patient has a question for you about his or her illnesses or history? You should understand that it is okay to tell the patient that you do not know. A good rule of thumb for your entire career in medicine is that making something up is never the right thing to do. Explain that you are still in the process of learning, and you will find out and get back to your patient. You use an interesting word when you say you are intruding. In some ways, your interview is an intrusion into intimate and private details of a person's life, but certainly your physical examination can be intrusive. Reading this made me immediately remind myself that this is truly what the privilege of being a physician (or physician in training) is all about. We put on a white coat, or merely introduce ourselves as doctors or student doctors, and patients share very private details with us. Additionally, they allow us to examine them to try and determine the etiology of their symptoms. It truly is an honor and a privilege, and sometimes it takes the novice to remind us of that. You are correct. It is an intrusion. However, the bigger picture is that the intrusion is necessary to get at the greater good, which is the successful diagnosis and treatment of your patients.
  • 28. It is hard for me as a young student to interview and examine patients. I feel like I am intruding into their personal world without having a lot to offer them, given my limited knowledge base. Do you have any advice? 27 So, let me congratulate you. You are truly a grounded person who has not yet forgotten the humanity in medicine. Please try to remember this feeling from time to time as you move forward in your career. Although we play a certain role as physician, our patients are people as are we. The physician who does not forget his or her roots, recognizes each patient as a unique individual, and remembers that this is an honor will serve his or her patients better. With time, your feelings will change. Some things will always seem a little strange but likely more "normal." This is neither good nor bad, just part of the process. I hope you can remember asking this question in the years to come.
  • 29. How Should I Deal With Gunners? 28 How Should I Deal With Gunners? How do you deal with a classmate who is a gunner? Is it something that I even need to worry about? Response from Sara Cohen, MD Fellow, Department of Physical Medicine and Rehabilitation, Harvard University, Boston, Massachusetts; Fellow, Department of Physical Medicine and Rehabilitation, VA Boston Healthcare System, Boston, Massachusetts During one of the first anatomy labs in medical school, I was approached by the professor. After he examined our dissection of the abdominal cavity, he reached his gloved hand deep into the cadaver and spoke to me for the very first time as he pointed at a blood vessel. "Sara," he said, "can you tell me the name of this artery?" I was pleased because I knew this one. I opened my mouth to answer the question, but before I could get a word out, my lab partner piped up, "The gastroepiploic artery." I was shocked. The professor had clearly addressed the question to me. He used my name and he was looking right at me. There was no way that could have been misunderstood. Why would my lab partner yell out the answer to a question that was obviously not directed at her before I even had a chance to speak up? It turned out that this was my first experience with a "gunner." The definition of "gunner" varies depending on who you talk to, but it generally refers to an especially competitive medical student. Every medical school class has at least 1 gunner, and usually many more than that. Some people use the term to refer to students who study much more than average and are especially concerned with grades. However, the term may also be used to refer to medical students who exhibit behavior that is either borderline unethical or even blatant cheating. Gunner behavior in the preclinical years that is borderline unethical includes (to name a few) hoarding study materials, making comments in front of professors that are meant to make the gunner look smarter and other classmates look unprepared, and dominating small group discussions. More malignant gunner behavior includes ripping pages out of textbooks in the library, sending out erroneous study materials to the class, or even cheating on examinations. As a medical student, you will almost certainly encounter a gunner classmate at some point. (If you don't, the gunner might be you!) Many specialties have become very competitive, which puts pressure on students who want to match in these specialties to be at the top of the class. If a gunner is making you feel unprepared or making it difficult for you to learn, the easiest approach is to avoid him or her in an academic setting. If you have a study group, make sure it only includes classmates who are conducive to your learning, rather than people who are trying to top you or make you feel insecure. Just because a classmate is your friend, that doesn't obligate you to study with that person as well.
  • 30. How Should I Deal With Gunners? 29 Another strategy is to set a good example. If you share your study materials with the class, it will encourage others to do the same. This is a subtle way to let gunners know that information should not be hoarded, and it is best for everyone to learn the material. If a gunner is making class very unpleasant by dominating the lecturers' time, and the subtle hints aren't working, the only option might be to approach him or her and explain your concerns. Most students don't want to be perceived as gunners, so pointing this out to the student might make him or her change. When you speak to your classmate, be nice and respectful when you address this issue -- after all, this is someone you're going to be working with for the next several years and you don't want to make an enemy. Finally, if the gunner is outright cheating, it is your responsibility to report this to the appropriate authorities. Unfortunately, during the clinical years, gunner behavior often escalates, because grades are largely based on evaluations from the attending physicians who are observing you. A gunner may try to be the first person at work every day and the last to leave. He or she may try to leap in and do every available procedure or surgery, even on a patient who belongs to another student. I've heard of students who looked up the laboratory results on their co-student's patients to be ready if the other student dropped the ball. The best thing you can do in that situation is try to ignore the gunner's behavior and do the best job you can on your own patients. If you are very diligent and knowledgeable, that will be apparent to your attendings and residents. However, if the gunner's behavior is very disruptive, the best recourse may be to confront him or her. Keep in mind that all attendings and residents were once medical students, and they're often able to recognize gunner behavior. Although the gunner may impress some attendings, others will be turned off by attempts at showing off. You may be gratified to discover that your attending dislikes your gunner co- student as much as you do. Although gunners are a common occurrence in medical school, there's no reason for them to make your life miserable unless you let them. Work hard and learn as much as you can and take gunners for what they are: harmless distractions.
  • 31. Are there any advantages to working outside medicine before you begin medical training? 30 Question Are there any advantages to working outside medicine before you begin medical training? Response from Sarah Bernstein, MD Resident, Department of Obstetrics and Gynecology, St. Luke's- Roosevelt Hospital, New York, NY Before I started medical school, I worked as a waitress for a year. It was mainly a way to make ends meet while I finished my premed requirements, but it turned out to be a very beneficial experience and one that has definitely affected my career as a physician. Approaching a table of customers is very similar to approaching patients. You have to read your customers; be approachable, friendly, and professional; and then do your best to serve them well. You need to work well in adverse circumstances: If the chef forgets their meal, a free drink is always helpful! You also need to make everything accessible and understandable to the everyday person. Back in those days, I often found myself describing the composition of a beurre blanc sauce; now, it is the warning signs of preeclampsia and preterm labor that I must explain. The endurance developed by long nights on my feet carrying trays of food up and down stairs certainly helped as well. Perhaps the most important lesson that I learned was to always check in on your customers. A customer can be content at one moment and then ready to throw the spaghetti at you the next, just as a patient can move from stable to crashing in a matter of seconds. You may ask whether I'm advocating that all medical students and residents join the food service industry before becoming physicians. Definitely not! I do think, however, that life experiences, regardless of their relevance to medicine, are very helpful in residency. Older residents who may have worked or traveled for a few years generally approach residency with a very different perspective; they understand that it is a very extreme and temporary situation. Many older residents also enter residency with a spouse, family, and maybe even a dog. They are challenged with budgeting their time and not neglecting their loved ones. On the plus side, they are often able to maintain a better life balance with the love and support of their families. No matter where you are in life, residency will be some of your most challenging years. Once you begin, there is very little time for more than just medicine, so any life experience that you bring from the years before training will be incredibly helpful.
  • 32. Once I start practicing in the "real world," can I still change specialties if I feel that I made a mistake? Or will I be stuck in that career, no matter what? 31 Question Once I start practicing in the "real world," can I still change specialties if I feel that I made a mistake? Or will I be stuck in that career, no matter what? Response from Geoffrey A. Talmon, MD Assistant Professor, University of Nebraska Medical Center, Omaha, Nebraska A friend of mine during residency had switched to pathology after being a rural family physician for many years. When I was a frustrated intern, I often asked him why he had chosen to go back to residency; his wife had a successful career, his children were all in elementary school, and he had already "paid his dues" as a resident. He said that in his previous practice, he was confident in handling common problems but often referred complex and interesting patients to specialists. Over time, he discovered that making one difficult diagnosis was more fulfilling than treating 10 patients with hypertension. Now in a pathology practice, he reiterates that although the move was initially hard for him and his family, he has found tremendous professional and personal gratification in his work and does not regret the switch. The perceived dogma is that being a physician represents a state of terminal vocational choice. Although it is not uncommon for residents to change specialties, many doctors believe that it is "too late" to alter their career path after entering practice, partly due to their substantial investment in training. Few (if any) residency programs broach the topic with their trainees. Practicing physicians may be reticent to discuss a partner's departure from their practice, even if the change had nothing to do with group dynamics. A lack of information perpetuates the notion that doctors are stuck with their initial career choice. The reality is that changing one's specialty is not unprecedented. In fact, as more Generation X'ers and Millennials populate medicine (with the increased tendency to job-shop), switching specialties likely will become even more common. Because there are few studies on this phenomenon and because mentors may be difficult to find, you may feel as though you are going where no one has gone before. For obvious reasons, making the decision to try a new field after being in practice is substantially more complex than when one was a resident. Re-entering training (with the attendant staff-resident hierarchy) may be less palatable. Relocation to a teaching hospital in a larger city may be necessary, placing strain on spouses with careers and families with older children. Further, the switch will engender a substantial salary decrease for several years, which may require special financial planning or a lifestyle change. Obviously, the concept requires careful discussion with and assistance from your family. Despite all of these potentially contentious issues, the ultimate factor to consider is your long-term happiness. Many other physicians have made these situations work, sometimes through their own creativity.
  • 33. Once I start practicing in the "real world," can I still change specialties if I feel that I made a mistake? Or will I be stuck in that career, no matter what? 32 Which new specialty to enter may or may not be obvious to you. Perhaps you have always had a "second choice" in the back of your mind since medical school. Regardless, it is important to consider several issues: What is the primary source of dissatisfaction with your current specialty? Which aspect of medicine interests you more? For example (as with my friend), you may find that you enjoy diagnostic challenges more than urgent care or health maintenance. Perhaps emergent care provides greater stimulation than long-term treatment of chronic illnesses. Do you want to do more procedures than your current specialty involves? Perhaps your most serious concerns lie with work hours and scheduling. Whatever the reason, this decision represents a substantial psychosocial investment, and it is imperative that the new field you enter is professionally and personally satisfying. Much like the first time you sought a residency, research is important. Spend time talking to multiple people from different types and sizes of practices about their experience and views. Specifically, determine the employment outlook and potential practice settings that would be available when you have completed training. To this end, it is beneficial to get a feel for the future of the specialty: What issues will you face in practice (economic liability, political pressures, the impact of mid-level practitioners, etc)? Length of training is also a consideration. Starting a 1- or 2-year fellowship is obviously less complicated than starting a 6-year surgery residency. Note that residencies may be willing to grant exemptions from a preliminary or intern year due to your experience, so be sure to inquire. When applying to training programs, be prepared not only to justify your decision but also to demonstrate that you have a good comprehension of the specialty and the previously mentioned issues associated with switching. Most program directors with whom I have spoken see the benefit and value of having an "uber-experienced" intern on service and on the educational team. Their primary concern is to maximize the training of the candidate and their contemporaries. Directors wish to ensure that you are making an informed, contemplated choice and are prepared to accept being a trainee again. Be certain to explicitly articulate your reasons for changing, and ask interviewers whether they believe that their program will provide what you are seeking. This reinforces to each person that you understand precisely what you want. One benefit of being a "nontraditional resident" is that you may be exempt from the traditional application process. This allows you to receive offers and accept a position outside of the Match, which affords a modicum of flexibility, as it is possible to enter residency at a time other than July 1. Once in training, continue to critically evaluate the new specialty that you have chosen. Taking into account the special challenges that are part and parcel of being a resident, is this new path giving you what you expected? Do not hesitate to discuss any perceived issues with your program director as soon as possible. Their advice is just as applicable to you as to traditional first-year residents. In summary, the process of changing specialties after residency is more complex than it is during training, but it likely will become more commonplace in coming years. Research is the most powerful tool to help you make informed decisions. The psychosocial and financial ramifications are not insurmountable, and the increased satisfaction may be worth the cost.
  • 34. Once I start practicing in the "real world," can I still change specialties if I feel that I made a mistake? Or will I be stuck in that career, no matter what? 33 Question I will soon be entering my fourth year of medical school, and I just learned that I've been assigned to a surgical rotation of which the surgeons are known to have very high standards and the failure rate for students is very high. However, I've decided not to swap rotations. How should I prepare myself for it? Response from Ted Melnick, MD Attending Physician, Department of Emergency Medicine, North Shore University Hospital, Manhasset, New York The surgical culture is very demanding and can intimidate the unprepared student. I remember struggling through my third-year surgical clerkship and being quite apprehensive of my required fourth-year surgical subinternship. However, I was surprised to discover that my third-year clerkship had prepared me well for the subinternship. As a fourth-year student, the skills that you have gained over the previous year can make you a valued member of the patient care team. The expectations of a final-year medical student are much different from those of the inexperienced, timid third-year student. You have a different knowledge base; you hopefully have some procedural skills; and you understand the workings of the hospital. The mere fact that you are already thinking ahead and have decided to accept the challenge of a more "difficult" rotation will put you in a better position for success. The overwhelming amount of work expected of the surgical resident can be lightened by a helpful, hardworking fourth-year medical student. If you are able to reduce a resident's workload, he or she will likely think positively of you and will provide positive feedback to the clerkship director. Here are a few guidelines to follow in order to survive, pass, and maybe even flourish on your fourth-year surgical rotation. First, arrive on time. Surgical rounds occur as a team. If you are late, the whole team will be waiting for you and will be unable to round on time. In fact, if you arrive a few minutes early every day, you will stand out as someone who understands teamwork and is there to help. Second, don't draw negative attention to yourself. Especially in the operating room (OR), don't speak unless spoken to. Although this sounds authoritarian, remember that surgery requires incredible dexterity and concentration. Asking a question at an inopportune time in the OR may distract the surgeon at a critical point in the procedure that he or she is performing. This also holds true on rounds. Time is limited there, so the team is trying to accomplish as much as possible as efficiently as they can. If you draw attention to yourself on rounds for anything but contributing to getting the job done, you may leave a bad impression.
  • 35. Once I start practicing in the "real world," can I still change specialties if I feel that I made a mistake? Or will I be stuck in that career, no matter what? 34 Third, help with floor work. An overworked surgical intern will sing your praises to his or her seniors if you help with some of the floor work, allowing him or her to get home a little earlier or to spend more valued time in the OR. You may want to read and carry The Surgical Intern Pocket Survival Guide.[1] This small reference offers all sorts of pearls that will come in handy while taking care of routine floor work and charting. Fourth, familiarize yourself with the book, Surgical Recall.[2] This book is indispensable. It provides just about any question that a surgeon has ever used to "pimp" a medical student. Find out which procedures you will be scrubbing for the next day, and read that section beforehand. You should also review anatomy relevant to the procedure in your first-year anatomy textbook. You can even go the extra mile by familiarizing yourself with the procedure in a surgical textbook. To the attending and senior residents, you will appear prepared, interested, and dedicated to the patient. Fifth, sleep and eat whenever you can. This statement is true throughout your clinical years in medical school. However, it is even more important on time-intensive rotations, such as surgery. A little nutrition and rest can make you much happier and more productive.
  • 36. Once I start practicing in the "real world," can I still change specialties if I feel that I made a mistake? Or will I be stuck in that career, no matter what? 35 Question The hospital environment is so drab and depressing. How can I stay motivated when I have to spend so much time there? Response from Daniel Egan, MD Attending Physician, Department of Emergency Medicine, St. Luke's- Roosevelt Hospital Center, New York, NY Ahh... We have all been there. My guess is that you are in the midst of your clinical rotations and you feel like all you do is wake up in the morning, go to the hospital, leave, sleep, and start the cycle again the next morning. Well, my friend, we have all had those feelings and wondered how to keep on going. There is no magic answer, but there are a couple of key elements to maintaining your sanity throughout medical school. The most important thing that I kept telling myself during my clinical rotations was that everyone in the class ahead of me got through it. Additionally, many of your classmates who have already completed the rotation that you are on have survived it. To me, that was motivating: They had endured, and so could I. It is a little reality check as you try to keep your eyes open during hour 24 in your surgical clerkship. I also kept telling myself throughout my medical school clerkships -- and especially residency -- that I could do anything for a fixed period of time. In residency, we changed rotations every month, so I would repeatedly say, "I can do anything for 4 weeks." But you are correct: The hospital environment can seem drab and can be depressing. Therefore, it is critical for you to maintain a life outside of the hospital. We all know that it is possible to eat, sleep, and dream medical school. There is so much to learn and there are not enough hours in the day. However, you need to remember that you are a person who had a life and interests before you got there. I have written about this before, but I often remember something that a dean told us students early in our education: Even if you spend 24 hours a day studying and trying to learn, you will still never know everything. So make a schedule. Make an agreement with yourself. Fit outside activities into your day (even if that means just an hour at the gym each evening). Consider it as mandatory as prerounding on your patients. Consider it essential to your health and well-being. There is a certain time of year (coincidentally about the same time as when you submitted your question) that seems to predispose students and also residents to feelings of "drabness" and depression. You wake up in the morning and it is dark outside. You leave the hospital to go home and it is dark outside. The rest of the world has enjoyed a full day of sunlight while you roamed the halls of the hospital and only caught a glimpse of the sun if you went into a patient's room. One of the coping mechanisms for this is to talk about it. Go out to dinner with your classmates and share your feelings. Of course, try not to talk only about work. Sometimes a nice, long venting session can do a lot of good; things are put in perspective and you recognize that you are not alone.
  • 37. Once I start practicing in the "real world," can I still change specialties if I feel that I made a mistake? Or will I be stuck in that career, no matter what? 36 This pattern will continue for the next several years as you go through residency, so developing distractions and coping mechanisms now will only help later. Perhaps the most important step that you can take is devoting at least part of your day to a nonmedical activity that you enjoy. This can provide the motivation for getting through the day. For me, that activity was participating in a music group. Although we didn't rehearse every day, that 1 night a week of doing something that I love helped me to get through those very long other 6 days. Keep a novel at your bedside. Spend even just 20 minutes before going to sleep reading something completely unrelated to a medical journal or textbook. Make a few phone calls to friends who are in other fields. Read the newspaper. And remind yourself regularly that there is a whole world out there that you will soon be able to embrace.
  • 38. I am starting my residency soon, and I am worried about being prepared. What resources should I bring to the hospital? How will I know what to do when I'm alone during night call? 37 Question I am starting my residency soon, and I am worried about being prepared. What resources should I bring to the hospital? How will I know what to do when I'm alone during night call? Response Sohil H. Patel, MD Sohil H. Patel, MD, Resident, St. Vincent's Hospital, New York, New York Beginning your intern year is daunting. I know of few other times when one's level of responsibility changes as dramatically as when a medical student becomes an intern. In some respects, the transition requires an understanding that the first time through many new experiences, you may be slow and you may make mistakes. There are certainly ways to minimize such mistakes and, most importantly, to learn from them so that they are not repeated. As a subintern, I lumbered around in my short white coat with pockets brimming full of "survival guides," including my personal digital assistant, a pharmacopoeia, my stethoscope, a reflex hammer, various scraps of paper and notecards with illegible notes and phone numbers, and maybe a flattened granola bar or two. Early in my subinternship, I was called to see a patient with abdominal pain and lower gastrointestinal (GI) bleeding. My thoughts immediately became as scattered as the contents of my white coat. In fact, my first 3 thoughts were: Is this an emergency? Should I be worried about colon cancer or angiodysplasia? What are those other causes of lower GI bleeding that I memorized for the boards? As I pondered these deep questions, my resident thankfully arrived and ordered me to check vital signs and do an examination. Only afterward did I look through the various resources in my white coat and find at least 3 resources with the same algorithm for dealing with GI bleeds. So, my first piece of advice is: know what's in your white coat! If you buy a survival guide (and I recommend the Washington University series), read through it before you put it in your white coat. Use only 1 survival guide, and become very familiar with it. Likewise, use either a personal digital assistant or a pharmacopoeia or an online pharmacology Website (if your hospital has enough computers), and learn how to use whichever resource you choose. I also found it helpful to carry a few blank note cards. Every day, I jotted down important phone numbers, or room codes, or simple instructions on how to get things done in the hospital. My notes were a mess, so each night I would rewrite them neatly on a new note card.
  • 39. I am starting my residency soon, and I am worried about being prepared. What resources should I bring to the hospital? How will I know what to do when I'm alone during night call? 38 Other white coat essentials include a stethoscope, a (functioning) pen light, a prescription pad, your sign- out, and at least 2 black ink pens. For internal medicine, the Mass General Pocket Medicine book is an excellent resource. If you can fit a snack somewhere in your pockets, that always comes in handy. Recognize Emergencies This is probably the most important skill learned during intern year. When a nurse calls you about a patient, particularly early in your intern year, you should try your best to always see the patient. See for yourself what types of calls and complaints are emergent and which are non-emergent. When you see a patient, the best way to determine whether the problem is an emergency is to talk to the patient and get a set of vital signs. Then, compare these findings with the patient's baseline findings. These 2 simple steps will provide critical information about a patient's neurological and cardiopulmonary status. Certain nursing calls and patient complaints will mandate that you see the patient. Chest pain, shortness of breath, acute abdominal pain, hypotension, mental status change, new neurological deficit, GI bleed, and new fever all qualify. Learn which causes and consequences of such complaints are emergent, and direct your history, physical examination, and management accordingly. Thus, if a patient complains of chest pain, your first objective is to rule out a myocardial infarction, pulmonary embolism, or aortic dissection. Only after doing so should you explore whether the chest pain is actually musculoskeletal in nature. Finally, especially early on, accept a low threshold for calling your senior resident about all suspected emergencies. Arm Your Brain The more medicine you know, the more you will enjoy and learn from your experiences as an intern. Unfortunately, the fourth year of medical school is a well-known cause of brain atrophy. Thus, I found it helpful to brush up before starting intern year. Review the most high-yield, commonly encountered diseases in your specialty, and know them well (including presentation, diagnosis, and treatment). You may also benefit from reviewing school textbooks and study aids. At a minimum, every new intern should know how to read an electrocardiograph, interpret an x-ray, and understand a blood gas result. Prepare Yourself Mentally Anxiety naturally accompanies unfamiliar situations, particularly when your decisions will affect someone else's health. Avoid adding more stress whenever possible. A common stressor is getting yelled at by a superior because you made a mistake. It is important to recognize 2 components to the message.: The first is the yelling, which is mostly a reflection of your senior's (probably somewhat dysfunctional) personality; the more this is ignored, the better. The other component is the actual message, which often contains very useful advice to avoid making the same mistake in the future. This second component deserves your full attention. Other ways to avoid needless stress are as follows:
  • 40. I am starting my residency soon, and I am worried about being prepared. What resources should I bring to the hospital? How will I know what to do when I'm alone during night call? 39  Try to build a good rapport with the support staff, and never engage in petty arguments with them;  Know your limits with difficult patients and recruit the help of social workers when needed;  Build good relationships with other residents;  Don't be afraid or embarrassed to ask for help from anyone (especially nurses, who are often your best friend on a busy call night); and  Do not constantly remind yourself how terrible an intern's life is. If you direct your thoughts to your patients and the amount of good you are doing for them, the intern year can be an incredibly rewarding experience.
  • 41. Twelve Steps for Choosing a Specialty 40 Twelve Steps for Choosing a Specialty Question I need to declare a medical specialty before long, but I have so many conflicting feelings and thoughts about various clinical areas. How can I make the right choice? Response from Anne Vinsel, MS, MFA Project Administrator, Graduate Medical Education, University of Utah Medical Center, Salt Lake City, Utah It's time for fourth year students to get serious about choosing their specialty area. Some of you are lucky, and everything lines up: you know which clinical area interests you most, your board scores and grades/letters are all in the correct range, and you have helpful professors on your side. For you, it's just a matter of doing the paperwork on time. You can stop reading here. But I know there are many others of you out there who aren't sure what specialty to choose. Or, you're torn between 2 or 3 specialties. Or you know what you don't want but aren't sure what you do want. Or you know what you want, but aren't sure if your qualifications are strong enough. Read on! If you're stuck, here's a decision tree to follow: 1. Find or make a list of all the specialties available directly after medical school (ie, skip fellowships). 2. Cross off the ones you definitely don't want. You don't need a string of reasons beyond the fact that you simply can't see yourself doing it long term. 3. Perform a Google™ search with the phrase "choosing a medical specialty." When I tried it, I got about 89,800,000 entries. Set a timer for no more than 1 hour and browse through the first several pages. Take some of the "what specialty are you?" quizzes. If nothing else, they will give you some ideas and possibly make you think about specialties you haven't explored. You can safely avoid making an exact ranking of specialties at this point. Just see which specialties you seem to be most suited to and which you should rule out. 4. Now, list several specialties you can see yourself doing long term, no more than 6. 5. Research those specialties in your institution. Go to the departments and make friends with the residency program coordinators. If you haven't already done so and haven't rotated in the program, arrange to shadow a faculty member for a day. Talk with 1 or 2 residents and check out the pros and cons of the specialty. Finally, ask the program coordinator if your board scores would be in a competitive range. Most program coordinators won't share their board score cut-off, but they likely would tell you if your scores are within range. 6. Narrow your list to 2 or 3 specialties. Now, and only now, talk with family and friends. Tell them you're thinking of these specialties, and get their opinions. Listen hard, and get them to articulate the basis for their opinions.